BRS- Anatomy Course 1 Exam 2

Pataasin ang iyong marka sa homework at exams ngayon gamit ang Quizwiz!

Choose the appropriate lettered site or structure in this transverse magnetic resonance imaging through the middle of the palm of a woman's right hand (see figure) that matches the following description: A lesion of the median nerve causes paralysis of this structure.

The answer is A. This is the flexor pollicis longus, which is innervated by the median nerve.

Choose the appropriate lettered site or structure in the radiograph of the elbow joint (see figure) and its associated structures to match the following description: A lesion of the radial nerve would most likely cause paralysis of muscles that are attached to this area.

The answer is D. The olecranon is the site for insertion of the triceps brachii, which is innervated by the radial nerve. When the olecranon is destroyed, the triceps brachii is paralyzed. (less)

Choose the appropriate lettered structure in the radiograph of the bones of the hand (see figure).Which is the site of attachment of the muscles that form the thenar eminence?

The answer is E. The base of the proximal phalanx of the thumb is the site of attachment for the flexor pollicis brevis, which, along with the opponens pollicis, forms the thenar eminence. It is also the site of attachment for the adductor pollicis brevis. (less)

Over a period of several months, a 30-year-old woman of slight body-build experiences increasing levels of pain running down the medial side of her right forearm and hand and wasting of the small muscles in that hand, especially noticeable in the medial aspect of the hand. Thorough medical examination and testing reveals abnormal narrowing of the apex of the right axilla causing compression of the lower trunk of the brachial plexus. Which of the following conditions best explains the unusually small apex of the axilla?

The answer is a bone callus formation on the clavicle. The apex (upper end) of the axilla is formed by the clavicle, upper border of the scapula, and outer border of the first rib. At this location, the brachial plexus (especially the lower trunk) and/or the subclavian vessels may be compressed between the bones as they enter the upper limb, especially in the interval between the clavicle and the first rib. The clavicle is the most commonly fractured bone in the body. Callus formation during repair may cause a narrowing of the apex of the axilla and compression of the neurovascular structures therein, especially if the fracture were at the more proximal end of the clavicle. The lesser tuberosity of the humerus is not a component of the apex of the axilla or any of the walls of the axilla. The upper border of the scapula, not the coracoid process, partly forms the apex of the axilla. Further, expansion of the tip of the coracoid process would be directed away from the apex. The thoracic outlet is formed by the T1 vertebra, medial border of the first rib and cartilage, and superior border of the manubrium sterni. The thoracic outlet and the apex of the axilla are closely neighboring spaces with important relations to the brachial plexus and subclavian vessels. Narrowing of either may produce symptoms similar to those described in this case. The pectoralis minor is a component of the anterior wall of the axilla, and is located below the apex of the axilla. (less)

A 38-year-old baseball pitcher felt a sharp pain in his shoulder when he threw the ball very hard. After a visit by the pitching coach, he threw another pitch but he told the catcher that the pain was unbearable. He left the game and went to the dressing room where he was examined by the team's orthopedic surgeon, who detected tenderness near the greater tubercle of the humerus. An MRI taken the next day revealed a tear in the pitcher's rotator cuff. Which of the following statements is most likely based on this finding?

The answer is acute rupture of the supraspinatus part of the rotator cuff. This is the most common cause of a rotator cuff injury. Repetitive overhead motions of the upper limb can impinge the supraspinatus and/or its tendon between the greater tubercle of the humerus and the osseofibrous coraco-acromial arch. This causes degenerative changes (attrition) that ultimately result in a sudden tear of the supraspinatus tendon. Acute rupture of the subscapular tendon is incorrect. This muscle inserts on the lesser tubercle, and rupture occurs less often than supraspinatus tears. Acute tear of latissimus dorsi tendon is incorrect. This muscle is not part of the rotator cuff group of muscles. Acute tear of teres major tendon is incorrect. This muscle is not part of the rotator cuff group of muscles. (less)

You are a treating a 3-year-old girl who is holding her left elbow and is clearly in pain. The mother states that her daughter tried to cross the street and a car was coming and she had to acutely lift the child by acutely jerking her left arm up and bringing her back on the sidewalk. What is your likely diagnosis?

The answer is annular ligament tear. This condition is referred to as "pulled elbow" or "nursemaid's elbow." The history is very typical and is as described. The sharp jerking movement on the limb tears the distal attachment of the annular ligament from the radius and the radial head moves distally. (less)

A 10-year-old boy falls off his bike, has difficulty in moving his shoulder, and is brought to an emergency department. His radiogram and angiogram reveal fracture of the surgical neck of his humerus and bleeding from the point of the fracture. Following this accident, the damaged nerve causes difficulty in abduction, extension, and lateral rotation of his arm. Cell bodies of the injured nerve involved in movement of his arm are located in which of the following structures?

The answer is anterior horn of the spinal cord. The (injured) axillary nerve contains GSE fibers whose cell bodies are located in the anterior horn of the spinal cord, and these GSE fibers supply the deltoid and teres minor muscles. The axillary nerve also contains GSA and GVA fibers, whose cell bodies are located in the dorsal root ganglia, and sympathetic postganglionic fibers, whose cell bodies are located in sympathetic chain ganglia. The lateral horn of the spinal cord between T1 and L2 contains cell bodies of sympathetic preganglionic fibers. The posterior horn of the spinal cord contains cell bodies of interneurons. (less)

A 23-year-old man was injured in a motor vehicle accident and X-rays confirmed a displaced distal radius fracture in his left forearm. Upon examination, the patient exhibits weakened pronation, weakened flexion of the index and middle fingers at the distal interphalangeal joints, and weakened flexion of the interphalangeal joint of the thumb. When asked to make the "okay" sign (make a circle with the thumb and index finger), the patient is unable to make a round circle, producing a "collapsed circle" on the affected hand (see photo). No areas of sensory loss are detected. Which nerve is most likely damaged?

The answer is anterior interosseous nerve. The anterior interosseous nerve is a branch of the median nerve in the distal part of the cubital fossa, and it courses distally on the interosseous membrane. It supplies the deep forearm flexors, including the flexor digitorum profundus of digits 2 and 3, the flexor pollicis longus, and the pronator quadratus. Loss of this nerve would cause weakness in pronation due to denervation of the pronator quadratus. This injury would also result in inability to flex the distal interphalangeal joints of the index and middle fingers and the interphalangeal joint of the thumb due to denervation of the flexor digitorum profundus and flexor pollicis longus, respectively. This deficit would lead to the collapsed "O.K. sign" indicated in the photo. Because compromising the anterior interosseous nerve would not result in any cutaneous sensory deficits, it is this nerve that was most likely damaged by the displaced end of the left radius. Deep branch of the radial nerve is incorrect. The deep branch of the radial nerve arises from the radial nerve in the cubital fossa, dives deep to pierce the supinator muscle, and supplies all the muscles in the posterior compartment of the forearm. Damage to this nerve would lead to "wrist drop" in the patient due to the flexors of the forearm being unopposed and an inability to extend at the wrist. Superficial branch of the radial nerve is incorrect. The superficial branch of the radial nerve arises from the radial nerve in the cubital fossa. This nerve is entirely a sensory nerve, supplying the dorsum of the hand and fingers. Because the patient has motor deficits, this nerve was not damaged by the displaced radius. Median nerve proximal to the carpal tunnel is incorrect. The median nerve proximal to the carpal tunnel would not be injured because damaging this nerve would lead to a significant sensory loss over the thumb and the adjacent two and a half fingers in addition to motor loss to the thenar eminence (opponens pollicis, abductor pollicis brevis, and flexor pollicis brevis muscles) and the first two lumbrical muscles. So, the median nerve was not injured proximal to the carpal tunnel. Recurrent (thenar) branch of median nerve is incorrect. The recurrent (thenar) branch of the median nerve innervates most of the thenar muscles, including the abductor pollicis brevis, opponens pollicis, and superficial head of the flexor pollicis brevis. Cutting this nerve would lead to atrophy of the thenar muscular complex, a condition known as "ape hand." In this condition, the patient would not be able to oppose the thumb and second finger. Though this nerve has no sensory distribution, it is unlikely that it was involved due to the weakness when pronating the forearm. (less)

A 3-year-old girl is brought to the emergency room holding her right arm with the elbow flexed and the forearm pronated. She refuses to move her arm and complains her elbow "hurts a lot." Her mother reports they were holding hands and running in the park when the child tripped. The mother pulled on the child's hand to prevent her from hitting the ground. Given the nature of this injury and the age of the patient, what structure is most likely damaged?

The answer is anular ligament of radius. The anular ligament of the radius encircles and holds the head of the radius in the radial notch of the ulna. This ligament enables pronation and supination of the forearm. However, the head of the radius can be pulled distally out of this anular ligament resulting in a subluxation or dislocation of the radial head, which is frequently called "nursemaid's elbow." This injury is often seen in children, particularly girls, between the ages of 1 to 3 years old. It occurs when an extended arm is pulled, commonly during a fall, and the individual holding the hand does not let go, as reported in this case. Subluxation and dislocation of the radial head are also seen when the child is swinging while being held by the hands. Interosseous membrane of the forearm is incorrect. The interosseous membrane of the forearm connects the interosseous borders of the radius and ulna. This membrane would not be injured in this type of the injury, especially because the patient complains of pain in the elbow region. Quadrate ligament is incorrect. The quadrate ligament passes from the distal margin of the radial notch of the ulna to the neck of the radius. Damage to this ligament is rare in this type of injury when the extended arm is pulled. Radial collateral ligament of elbow is incorrect. The radial collateral ligament of the elbow extends from the lateral epicondyle of the humerus to the anular ligament of the radius. This ligament would be injured in forced adduction of the elbow joint, which was not seen in this patient. Ulnar collateral ligament of elbow is incorrect. The ulnar collateral ligament of the elbow extends from the medial epicondyle of the humerus to the coronoid process and olecranon of the ulna. This ligament would be injured in forced abduction of the elbow joint, which was not seen in this patient. This ligament is also frequently injured in athletes who use an overhead throwing motion. When the athlete releases the ball during their throwing motion, the ulna is pulled from the humerus, which can damage this ligament. American baseball pitchers often need surgery to repair the ulnar collateral ligament of the elbow, which is commonly termed "Tommy John surgery." (less)

The police bring in a murder suspect who has been in a gunfight with a police officer. The suspect was struck by a bullet in the arm; his median nerve has been damaged. Which of the following symptoms is likely produced by this nerve damage?

The answer is ape hand. Injury to the median nerve produces the ape hand (a hand with the thumb permanently extended). Injury to the radial nerve results in loss of wrist extension, leading to wrist drop. Damage to the upper trunk of the brachial plexus produces waiter's tip hand. A claw hand and flattening of the hypothenar eminence or atrophy of the hypothenar muscles result from damage to the ulnar nerve. (less)

A 17-year-old boy is injured in an automobile accident. He has a fracture of the shaft of the humerus. Following this accident, the patient has no cutaneous sensation in which of the following areas?

The answer is area of the anatomic snuffbox. The superficial branch of the radial nerve runs distally to the dorsum of the hand to innervate the radial side of the hand, including the area of the anatomic snuffbox and the radial two and one-half digits over the proximal phalanx. The medial aspect of the arm is innervated by the medial brachial cutaneous nerve; the lateral aspect of the forearm is innervated by the lateral antebrachial cutaneous nerve of the musculocutaneous nerve; the palmar aspect of the second and third digits is innervated by the median nerve; and the medial one and one-half fingers are innervated by the ulnar nerve. (less)

A 21-year-old patient has a lesion of the upper trunk of the brachial plexus (Erb-Duchenne paralysis). Which of the following is the most likely diagnosis?

The answer is arm tending to lie in medial rotation. A lesion of the upper trunk of the brachial plexus results in a condition called "waiter's tip hand," in which the arm tends to lie in medial rotation because of paralysis of lateral rotators and abductors of the arm. The long thoracic nerve, which arises from the root (C5-C7) of the brachial plexus, innervates the serratus anterior muscle that can elevate the arm above the horizontal. The dorsal scapular nerve, which arises from the root (C5), innervates the rhomboid major. The medial side of the arm receives cutaneous innervation from the medial brachial cutaneous nerve of the medial cord. The adductor pollicis is innervated by the ulnar nerve. (less)

A patient who had wrist pain and a history of wrist injury underwent wrist MR arthrogram with the resulting image shown above. The image was obtained after an intra-articular injection of gadolinium-based contrast material (bright fluid) into the wrist (radiocarpal) joint. The contrast material flowed into (and distends) the cavity between the distal radius and ulna. This finding is consistent with a tear in the major supporting structure of the distal radio-ulnar joint, which is which of the following?

The answer is articular disc (triangular ligament, or triangular fibrocartilage). This disc binds the ends of the ulnar and radius together and is the main uniting structure of the joint. The articular disc separates the cavity of the distal radio-ulnar joint from the cavity of the wrist joint. (less)

A 17-year-old male football player suffers a shoulder injury and arrives at the ER 2 hours after the injury. The physician diagnoses a shoulder dislocation, and after administration of a local anesthetic solution, the doctor repositions the head of the humerus into the glenoid cavity of the scapula (reduction). No fractures are seen on X-rays. However, the patient displays weakness in abduction and external rotation at the shoulder. A loss of sensation is also noted at the superior and lateral aspects of the arm. What nerve was most likely damaged in this injury?

The answer is axillary nerve. The axillary nerve may be damaged in approximately one of seven shoulder dislocations. This nerve innervates the deltoid and teres minor muscles as well as supplying innervation to the skin overlying the deltoid in the superolateral aspect of the arm. Loss of innervation to the deltoid muscle would explain the weakness in abduction of the upper limb. The teres minor assists the infraspinatus muscle in external rotation of the shoulder. Median nerve is incorrect. The median nerve does not branch proximal to the elbow, and its sensory distribution is limited to the hand. The median nerve could not be responsible for the patient's motor and sensory deficits. Ulnar nerve is incorrect. The ulnar nerve can be damaged during shoulder dislocations; however, its sensory distribution is limited to distal to the wrist, and the first muscle it innervates is in the forearm (flexor carpi ulnaris). Therefore, the ulnar nerve could not be responsible for this patient's motor and sensory deficits. Radial nerve is incorrect. The radial nerve supplies motor innervation to the posterior compartments of the arm and forearm. Damage to this nerve would cause weakness in extending at the elbow and wrist joints. The radial nerve gives rise to the posterior cutaneous nerves of the arm and forearm as well as the inferior lateral cutaneous nerve of the arm. However, it would not affect the superior lateral cutaneous nerve of the arm, arising from the axillary nerve, which was damaged in this patient. The radial nerve would also not affect abduction and external rotation of the shoulder. Musculocutaneous nerve is incorrect. The musculocutaneous nerve can be damaged during shoulder dislocations; however, this nerve supplies the motor innervation to the anterior compartment of the arm. Trauma to this nerve would lead to weakness in flexing the elbow and supinating the forearm. Its sensory distribution is limited to the lateral aspect of the forearm, so it was not the nerve damaged in this patient. (less)

A young man is brought to the emergency department after being mugged. He was stabbed in the shoulder after refusing to give his wallet to his assailant. If the stab wound lacerated the posterior humeral circumflex artery passing through the quadrangular space on the shoulder region, which of the following nerves might be injured?

The answer is axillary nerve. The axillary nerve runs posteriorly to the humerus, accompanying the posterior humeral circumflex artery through the quadrangular space and innervating the teres minor and deltoid muscles. None of the other nerves pass through the quadrangular space. (less)

Physical examination of a 45-year-old man who had been stabbed in the back of the shoulder shows a deep wound penetrating into the quadrangular space of the shoulder, causing bleeding from the severed blood vessels there. Which of the following neural structures is most likely damaged as well?

The answer is axillary nerve. The quadrangular space of the shoulder is an important passageway allowing the posterior humeral circumflex vessels and their companion axillary nerve to pass from the axilla to the posterior aspect of the shoulder. The neurovascular bundle runs across the surgical neck of the humerus to enter the quadrangular space. The space itself (sometimes termed the lateral axillary hiatus) is formed by four structures: teres major, teres minor, long head of the triceps, surgical neck of the humerus. The vessels contribute to the collateral network around the shoulder. The axillary nerve supplies the teres minor and deltoid muscles and a cutaneous area on the superolateral aspect of the arm (i.e., the skin overlying the lower aspect of the deltoid muscle). Musculocutaneous nerve is incorrect. The musculocutaneous nerve is a terminal branch of the lateral cord of the brachial plexus within the axilla. It supplies the anterior compartment of the arm and is not related to the quadrangular space. Lateral cord of the brachial plexus is incorrect. The lateral cord occupies a relatively lateral position in the axilla but is not related to the quadrangular space. The axillary nerve originates from the posterior cord of the brachial plexus. Radial nerve is incorrect. The radial and axillary nerves are the terminal branches of the posterior cord of the brachial plexus. The radial nerve runs through the radial (spiral) groove in the midshaft of the humerus to emerge through the lower triangular space, just below the quadrangular space, and enter the posterior compartment of the arm. Medial cutaneous nerve of the arm is incorrect. The medial cutaneous nerve of the arm is a branch of the medial cord of the brachial plexus. It is far removed from the quadrangular space. (less)

Hypothenar hammer syndrome is characterized by unilateral finger ischemia, caused by digital artery occlusions and is associated with repetitive striking of the palm. It has been postulated that repetitive striking of the palm or exposure of the palm to constant pressure or vibrational forces may lead to injury of the ulnar artery as it passes through Guyon's canal. Guyon's canal is located where?

The answer is between the hook of the hamate and the pisiform. The ulnar nerve and artery enter the hand through Guyon's canal, which is between the hook of the hamate and the pisiform. (less)

A 64-year-old man with a history of liver cirrhosis has been examined for hepatitis A, B, and C viruses. In an attempt to obtain a blood sample from the patient's median cubital vein, a registered nurse inadvertently procures arterial blood. The blood most likely comes from which of the following arteries?

The answer is brachial. The median cubital vein lies superficial to the bicipital aponeurosis and thus separates it from the brachial artery, which can be punctured during intravenous injections and blood transfusions. (less)

A 27-year-old baseball player is hit on his forearm by a high-speed ball during the World Series, and the muscles that form the floor of the cubital fossa appear to be torn. Which of the following groups of muscles have lost their functions?

The answer is brachialis and supinator. The brachialis and supinator muscles form the floor of the cubital fossa. The brachioradialis and pronator teres muscles form the lateral and medial boundaries, respectively. The pronator quadratus is attached to the distal ends of the radius and the ulna. (less)

A biomedical engineer would like to reconstruct the arm of a boy who underwent amputation to treat a life-threatening infection. In designing the prosthetic arm, the engineer will need to know that which of the following muscles flexes the elbow and is innervated by the radial nerve?

The answer is brachioradialis. The brachioradialis is innervated by the radial nerve and functions to flex the elbow. The flexor digitorum longus and extensor digitorum longus do not act in the elbow. The biceps brachii and brachialis muscles flex the elbow and are innervated by the musculocutaneous nerve. (less)

The physical therapist is testing the action of one of your patient's muscles by bringing the patient's upper limb away from her side and then asking the patient to abduct against resistance. What is the muscle being tested?

The answer is deltoid. The deltoid is the primary abductor of the upper limb past the first 15 degrees of abduction.

During a breast examination of a 56-year-old woman, the physician found a palpable mass in her breast. Which of the following characteristics of breast cancer and its diagnosis is correct?

The answer is dimpling of the overlying skin. Breast cancer may cause dimpling of the overlying skin because of shortening of the suspensory (Cooper's) ligaments and inverted or retracted nipple because of pulling on the lactiferous ducts. Polymastia is a condition in which more than two breasts are present. (less)

You are treating a patient in the ED who is supporting his right upper limb with his left hand. A radiograph of his shoulder is shown above. What is your diagnosis?

The answer is dislocated shoulder. The humeral head is not in its normal position in the glenoid fossa.

Yergason's test is a test for which a positive sign is likely to indicate rupture of the transverse humeral ligament. Such a rupture would be associated with which of the following?

The answer is dislocation of the tendon of the long head of the biceps brachii. This ligament holds the tendon of the long head of the biceps in the groove.

A 45-year-old secretary experienced "pins and needles" sensations (paresthesia) and pain in her right hand during the night. These sensations involved the palmar surface of her thumb and the lateral two and a half fingers. Her colleagues told her that she probably has carpal tunnel syndrome. Her family physician confirmed that this was the condition she was experiencing. She also told the physician that she was having some difficulty typing. On further examination, the physician detected some weakness of her right thenar muscles when compared with the left side. Otherwise motor functions were unimpaired currently. Which of the following statements best describes the most likely cause of the paresthesia experienced by the secretary?

The answer is distal (carpal) compression of the median nerve. This nerve is compressed as it passes through the carpal tunnel beneath the flexor retinaculum. It is the most common nerve impingement syndrome. This syndrome of nocturnal hand paresthesia and pain is caused by a variety of conditions, such as edema of chronic trauma and tenosynovitis, which produce impingement of the median nerve in the carpal tunnel. Compression of the ulnar nerve is incorrect. Compression of this nerve may occur at the wrist, where it passes through an osseofibrous tunnel (Guyon canal). The ulnar nerve supplies the medial one and a half digits and the hypothenar muscles. The symptoms described by the patient involved the lateral three and a half digits and the thenar muscles. Proximal trauma to the median nerve is incorrect. Proximal median nerve injury, resulting from a perforating wound of the elbow region, causes a loss of flexion of the proximal and distal interphalangeal joints of the 2nd and 3rd digits in addition to the symptoms resulting from distal compression of the median nerve. Injury of the radial nerve in the arm is incorrect. Although the radial nerve supplies no muscles in the hand, injury to it can produce serious disability to the wrist (i.e., "wrist-drop" resulting from paralysis of the extensor muscles). This injury would not cause the symptoms described by the patient. Compression of the radial artery at the wrist is incorrect and would not cause muscle weakness. (less)

A 24-year-old carpenter suffers a crush injury of his entire little finger. Which of the following muscles is most likely to be spared?

The answer is dorsal interossei. The dorsal interossei are abductors of the fingers. The little finger has no attachment for the dorsal interosseous muscle because it has its own abductor. Therefore, the dorsal interosseous muscle is not affected. Other muscles are attached to the little finger, thus they are injured. (less)

As part of a physical examination to evaluate muscle function in the hand, a physician holds the four fingers (digits 2 through 5) and asks the patient to spread their fingers, as shown below. What muscle(s) is/are the doctor testing?

The answer is dorsal interosseous muscles. The primary action of the dorsal interossei is to abduct the fingers, which is being tested in this photo. It is also important to remember the dorsal interossei insert into the extensor digital expansion, so these muscles work with the palmar interossei and the lumbrical muscles to extend the interphalangeal joints and flex the metacarpophalangeal joints. The nerve being tested in this patient, via abduction of the fingers, is the deep branch of the ulnar nerve. A mnemonic is "DAB", which stands for Dorsal interossei ABduct the fingers. Lumbrical muscles is incorrect. The lumbrical muscles arise off the flexor digitorum profundus tendons and insert into the lateral (radial) aspect of the extensor digital expansion. Due to their course and insertion, the lumbrical muscles extend the interphalangeal joints and flex the metacarpophalangeal joints. These intrinsic hand muscles are not being tested when a patient abducts the fingers against resistance. Palmar interosseous muscles is incorrect. The primary action of the three palmar interossei is to adduct the index finger, ring finger, and little finger toward the axis of the hand, which is the middle finger. The deep branch of the ulnar nerve supplies the palmar interossei. A mnemonic is "PAD", which stands for Palmar interossei ADduct the fingers. It is also important to remember the palmar interossei insert into the extensor digital expansion, so these muscles work in concert with the dorsal interossei and the lumbrical muscles to extend the interphalangeal joints and flex the metacarpophalangeal joints of the fingers. Flexor digitorum superficialis is incorrect. The flexor digitorum superficialis muscle flexes the proximal interphalangeal joint in digits 2 to 5, and influences the distal interphalangeal joint by binding the tendons of the flexor digitorum profundus muscle. Flexion of the proximal interphalangeal joints of the fingers is not being tested in this patient. Flexor digitorum profundus is incorrect. The flexor digitorum profundus muscle is responsible for flexion of the distal interphalangeal joint in digits 2 to 5; however, this action is not being tested in this photo. (less)

A 10-year-old boy falls off his bike, has difficulty in moving his shoulder, and is brought to an emergency department. His radiogram and angiogram reveal fracture of the surgical neck of his humerus and bleeding from the point of the fracture. The damaged nerve causes numbness of the lateral side of the arm. Cell bodies of the injured nerve fibers involved in sensory loss are located in which of the following structures?

The answer is dorsal root ganglia. Axillary nerve contains GSE, GSA, GVA, and sympathetic postganglionic GVE fibers. Cell bodies of GSA and GVA fibers are located in the dorsal root ganglia. Cell bodies of GSE fibers are located in the anterior horn of the spinal cord. Cell bodies of sympathetic postganglionic GVE fibers are located in the sympathetic chain ganglia, but cell bodies of sympathetic preganglionic GVE fibers lie in the lateral horn of the spinal cord. (less)

A patient bleeding from the shoulder secondary to a knife wound is in fair condition because there is vascular anastomosis around the shoulder. Which of the following arteries is most likely a direct branch of the subclavian artery that is involved in the anastomosis?

The answer is dorsal scapular artery. The dorsal scapular artery arises directly from the third part of the subclavian artery and replaces the deep (descending) branch of the transverse cervical artery. The suprascapular and transverse cervical arteries are branches of the thyrocervical trunk of the subclavian artery. The thoracoacromial artery is a short trunk from the first or second part of the axillary artery and has pectoral, clavicular, acromial, and deltoid branches. (less)

Your 17-year-old male patient was in a fist fight in a bar and now has pain along the medial side of his distal hand, along with flexion deformity (caused by volar angulation of the distal fracture fragment). A radiograph of his hand is shown above. What is your diagnosis?

The answer is fifth metacarpal fracture. When untrained people punch with a closed fist, the fifth metacarpal often fractures. This is known as a Boxer's Fracture.

Choose the appropriate lettered site or structure in the following radiograph of the wrist and hand (see figure). Destruction of the structure indicated by the letter E most likely causes weakness of which of the following muscles?

The answer is flexor carpi ulnaris. The hook of hamate and the pisiform provide insertion for the flexor carpi ulnaris.

As part of a physical examination to evaluate muscle function in the hand, a physician holds the proximal interphalangeal joint of his patient's index finger in the extended position and instructs him to try to flex the distal interphalangeal joint, as shown below. Which of the following muscles is the doctor testing?

The answer is flexor digitorum profundus (FDP). Flexion of the distal interphalangeal joint in digits 2 to 5 is produced by the FDP. The actions of this muscle are being tested in this illustration. Extensor indicis is incorrect. The extensor indicis extends the index finger (digit 2), which enables this finger to extend independent of the other fingers. Because the muscle arises from the distal third of the ulna and the interosseous membrane, it also acts to extend the hand at the wrist. The extensor indicis muscle is not involved in flexion of the distal interphalangeal joint, which is being tested in this patient. First lumbrical is incorrect. The first lumbrical muscle extends the interphalangeal joints of the index (second) finger and flexes the metacarpophalangeal joint of the same finger. The first lumbrical is an intrinsic hand muscle that arises off the tendon of the flexor digitorum profundus and inserts into the extensor expansion of the index finger. This muscle is not involved with flexion of the distal interphalangeal joint. First dorsal interosseous is incorrect. The primary movement of the first dorsal interosseous is abduction of the index finger. However, because it inserts into the extensor expansion, it also extends the interphalangeal joints of the index (second) finger and flexes the metacarpophalangeal joint of the same finger. This muscle is not involved with flexion of the distal interphalangeal joint. Flexor digitorum superficialis (FDS) is incorrect. The FDS acts at the proximal interphalangeal joint in digits 2 to 5 and influences the distal interphalangeal joint by binding the tendons of the FDP. However, when the proximal interphalangeal joint is held in extension, the influence of the FDS is eliminated, allowing testing of only the FDP. (less)

A patient with a deep stab wound in the middle of the forearm has impaired movement of the thumb. Examination indicates a lesion of the anterior interosseous nerve. Which of the following muscles is paralyzed?

The answer is flexor digitorum profundus and pronator quadratus. The anterior interosseous nerve is a branch of the median nerve and supplies the flexor pollicis longus, half of the flexor digitorum profundus, and the pronator quadratus. The median nerve supplies the pronator teres, flexor digitorum superficialis, palmaris longus, and flexor carpi radialis muscles. A muscular branch (the recurrent branch) of the median nerve innervates the thenar muscles. (less)

A 17-year-old man has pain and moderate swelling over the dorsomedial aspect and in the hypothenar area of his right hand after punching a locker over a dispute with his girlfriend. What is the most likely finding on an X-ray of his hand?

The answer is fracture of the fifth metacarpal bone. Fracture of the fifth metacarpal bone is called a "boxer's fracture" because this injury is often seen after an individual improperly punches a solid object with a clenched fist. The impact on the head of the fifth metacarpal causes the distal shaft of this bone to fracture. Skilled pugilists are trained to direct the impact of the clenched fist on the heads of the first and second metacarpals to avoid this type of injury. Dislocation of the fifth metacarpophalangeal joint is incorrect. This type of injury is rare, and a dislocation of the fifth metacarpophalangeal joint would have been detected by the physician on examination. Fracture of the triquetral bone is incorrect. This carpal bone is rarely fractured in this type of impact. Fracture of the proximal phalanx of the ring finger is incorrect. The impact of the clenched fist would have been localized to the head of the fifth metacarpal. Damage to the proximal phalanx of the ring finger would have occurred only if the fist was not clenched during impact. Fracture of the proximal phalanx of the little finger is incorrect. The impact of the clenched fist would have been localized to the head of the fifth metacarpal. Damage to the proximal phalanx of the little finger would have occurred only if the fist was not clenched during impact. (less)

A 19-year-old man was rollerblading when he caught his skate in a wide crack in the cement sidewalk. He said he fell on his open hand with his wrist extended and abducted. As his hand was very painful and slightly swollen, he went to the emergency department of a hospital. On examination, the physician noted that all wrist movements were painful, and there was tenderness in his anatomical snuffbox, but no unusual features were palpable. Radiographs of his wrist did not reveal a fracture. Because wrist movements were painful, the physician applied a light plaster cast. When the cast was removed 3 weeks later, wrist movements were still painful and the pain in the snuffbox was severe when the floor of the box was compressed with the thumb. A repeat radiograph revealed a decrease in the bone density of part of a carpal bone. The reduced bone density made the proximal part of the bone appear dark relative to the distal part of the bone and other carpal bones. Which of the following statements best describes the most likely cause of the man's continuing wrist pain?

The answer is fracture of the scaphoid. This bone is the most frequently fractured carpal. The pain occurs primarily on the lateral side of the wrist (snuffbox), especially during dorsiflexion and abduction of the hand. The fracture becomes obvious on radiographs approximately 2 to 3 weeks after the injury but can be seen on an MRI much earlier. By this time, bone absorption has occurred in the proximal fragment that has lost its blood supply. The nutrient artery enters the distal part of the scaphoid. The fracture becomes more obvious, partly because of the change in bone density of the proximal fragment. Dislocation of the lunate is incorrect. Anterior dislocation of this carpal is possible but uncommon. The lunate is squeezed out of position toward the palm and is apparent on physical examination. The physical examination did not reveal this abnormality. Fracture of the trapezium is incorrect and unlikely in this injury. Fracture of the hamate is incorrect because this bone is on the medial side of the wrist. Fracture of the pisiform is incorrect because this bone is on the medial side of the wrist. (less)

On his downswing, an amateur golfer strikes the hard earth with his club and feels pain in his right wrist. During a subsequent physical examination, he complains of wrist pain that is exacerbated by gripping, displays point tenderness in his medial wrist, and complains of numbness and weakness in his pinky finger (fifth digit). What carpal bone, identified by the white arrow on the given X-ray, is most likely fractured in this patient?

The answer is hamate. The hamate bone is identified by the white arrow in the given X-ray. This bone resides in the medial (ulnar) aspect of the distal row of carpal (wrist) bones. Stress fractures can occur to this bone, particularly within its hook, which appears as a radiodense oval on this radiograph. This type of fracture is frequently seen in golfers due to the positioning of the proximal aspect of the golf club within their grip. A fracture of the hamate results in pain, which is exacerbated by gripping and point tenderness in the skin overlying the bone. The numbness and weakness in the medial aspect of the hand, seen in this patient, are due to impingement of the ulnar nerve. Remember, the eight carpal (wrist) bones form two rows that contain four bones each. The proximal row (from lateral to medial) is composed of the scaphoid, lunate, triquetrum, and pisiform. The distal row (from lateral to medial) is composed of the trapezium, trapezoid, capitate, and hamate. To remember the carpal bones as listed, some students use the mnemonic: "Some Lovers Try Positions That They Can't Handle". In a standard posteroanterior plane film, the medial aspect of the wrist can be deceiving to the untrained eye. Here, the pisiform overlies the triquetrum, and the seemingly single radiopacity might not be distinguished as two separate bones. Instead, the distinctive hook of the hamate bone may be mistaken for the pisiform. Capitate is incorrect. The capitate (L: head) is a head-shaped bone that is the largest of the carpal bones. Its position is noted by its articulation with the third metacarpal distally. Lunate is incorrect. The lunate (L: moon) is a moon-shaped bone between the scaphoid and triquetral bones. It articulates with the radius proximally, and its position makes it the most commonly dislocated carpal bone. Pisiform is incorrect. The pisiform (L: pea) is a pea-shaped sesamoid bone that lies on the palmar aspect of the triquetrum bone. Scaphoid is incorrect. The scaphoid (G: boat) is a boat-shaped bone that is the largest of the carpal bones located in the proximal row. Its articulation with the radius proximally makes it the most commonly fractured carpal bone, especially when a person falls and impacts an abducted, outstretched hand. (less)

A 56-year-old avid tennis player is seeing you in the ED after a game in which he heard a popping sound and now has shoulder/arm pain. There is a ball-like swelling near the anterior distal part of his arm consistent with a Popeye deformity. You suspect the patient has which of the following?

The answer is has torn the long head of his biceps brachii. A Popeye deformity results from a torn long head of the biceps. It is associated with biceps tendinitis and is relatively common in tennis and baseball players. (less)

You suspect that your 61-year-old patient has ulnar canal syndrome. Which of the following would not be consistent with such a diagnosis?

The answer is inability to flex the distal interphalangeal joints of digits 4 and 5. The branch of the ulnar nerve that innervates the medial two tendons of the FDP arises from the ulnar near the elbow and thus the action of this muscle would not be affected by this syndrome, which involves compression of the ulnar nerve in Guynon's canal, at the wrist. (less)

An indoor soccer player runs into another player while running after the ball. She falls to the ground and fractures the medial epicondyle of the humerus. Which of the following symptoms might she present with when seeing a physician in the emergency department?

The answer is inability to sweat on the medial part of the hand. Fracture of the medial epicondyle of the humerus might injure the ulnar nerve, which supplies the skin of the medial side of the hand; thus, a lesion of the ulnar nerve would cause no cutaneous sensation and lack of sweating in that area. The muscles involved in abduction of the hand are the flexor carpi radialis and the extensor carpi radialis longus and brevis, which are innervated by the median and radial nerves, respectively. Carpal tunnel syndrome and thenar atrophy result from a lesion of the median nerve, whereas wrist drop results from a lesion of the radial nerve. (less)

A 7-year-old boy falls from a tree house and is brought to the emergency department of a local hospital. On examination, he has weakness in rotating his arm laterally because of an injury of a nerve. Which of the following conditions is most likely to cause a loss of this nerve function?

The answer is inferior dislocation of the head of the humerus. Inferior dislocation of the head of the humerus may damage the axillary nerve, which arises from the posterior cord of the brachial plexus, runs through the quadrangular space accompanied by the posterior humeral circumflex vessels around the surgical neck of the humerus, and supplies the deltoid and teres minor, which are lateral rotators of the arm. (less)

During a rushed evening in the ED you are asked to inject a mildly toxic chemotherapy agent into a patient's venous system. You quickly find a vessel in the patient's cubital fossa and inject the agent. Almost immediately the patient begins to complain of pain in her forearm and hand. You realize immediately that you erred. What did you most likely do?

The answer is injected the agent into a superficial ulnar artery. In approximately 3% of people, the ulnar artery traverses the cubital fossa superficial to the flexor muscles. It can inadvertently be mistaken for the medial cubital vein. (less)

An anesthesiologist administers an anesthetic solution into the axillary sheath of a 19-year-old college baseball player in preparation for repair of the ulnar collateral ligament of the elbow. After 5 minutes, the patient experiences numbness and paresthesia distal to the middle aspect of the arm; however, the medial aspect of the arm and elbow remain sensitive to pain. What nerve provides sensory innervation to the sensitive area and was not blocked by the anesthetic solution?

The answer is intercostobrachial nerve. The intercostobrachial nerve is the lateral cutaneous branch of the second intercostal nerve. As its name implies, it leaves the thorax by exiting between the second and third ribs (between the ribs = intercosto-) to supply cutaneous innervation to the axilla and medial aspect of the arm (brachium). In some instances, it may also supply skin distal to the elbow. The anesthetic solution would block all of the distal branches of the brachial plexus residing within the axillary sheath, thus sparing the intercostobrachial nerve. Long thoracic nerve is incorrect. The long thoracic nerve does not have a cutaneous distribution and provides only motor innervation to the serratus anterior. Because the long thoracic nerve arises from the ventral rami of C5-C7, it would not be affected by the anesthetic solution, especially if an occlusive tourniquet was utilized to retain the solution. Such a procedure is used to prevent spread of the anesthetic into the neck where it could affect the phrenic nerve and negatively affect respiration. Median nerve is incorrect. The median nerve does not branch proximal to the elbow, and its sensory distribution is limited to distal to the wrist. Furthermore, this nerve would be blocked by effective administration of the anesthetic reagent. Medial cutaneous nerve of the arm is incorrect. The medial cutaneous nerve of the arm arises from the medial cord of the brachial plexus, and it would be blocked by effective administration of the anesthetic solution into the axillary sheath. This nerve would also provide sensation to the area (medial aspect of the arm) that remains sensitive to pain, but it should be blocked by the drug. Ulnar nerve is incorrect. The ulnar nerve does not branch proximal to the elbow, and its sensory distribution is limited to distal to the wrist. Furthermore, this nerve would be blocked by effective administration of the anesthetic solution. (less)

A large dog knocked down a 79-year-old woman. Trying to ease her fall, she outstretched her upper limb. When she sat up, she noticed that her "wrist" (distal forearm) was deformed and very painful. She was taken to a hospital where the orthopedic surgeon said she had a Colles fracture. Which of the following statements about a Colles fracture is correct?

The answer is it is a fracture of the distal end of the radius. This is the most common type of "wrist fracture" in adults older than 50 years, especially in older women whose bones may be weakened by osteoporosis. It is fracture of the distal ulna is incorrect. The fracture commonly results from forced flexion of the hand is incorrect. The fracture usually results from forced extension of the hand, often the result of trying to ease the fall by outstretching the upper limb. The distal fragment of the fractured bone is displaced anteriorly is incorrect. The distal fragment is displaced posteriorly. Because of the way this alters the profile of the limb when viewed laterally or medially, this fracture is often called a "dinner fork deformity." In this fracture, the distal end of the radius is located distal to that of the ulna is incorrect. This is the normal condition. It is reversed in Colles fracture because of the fracture of the radius. (less)

A patient with a stab wound receives a laceration of the musculocutaneous nerve. Which of the following conditions is most likely to have occurred?

The answer is lack of sweating on the lateral side of the forearm. The musculocutaneous nerve contains sympathetic postganglionic fibers that supply sweat glands and blood vessels on the lateral side of the forearm as the lateral antebrachial cutaneous nerve. The musculocutaneous nerve does not supply the extensors of the forearm and the brachioradialis. This nerve also supplies tactile sensation on the lateral side of the forearm but not the arm and supplies blood vessels on the lateral side of the forearm but not the hand. (less)

A 42-year-old woman decided she wanted to learn how to play tennis. After she had practiced daily for about three weeks, she began to experience pain over the lateral region of her elbow. Her coach asked her to hold the tennis racket and extend her hand at the wrist. She felt no pain until he resisted the extension of her hand. When he asked her to pinpoint the site of the pain, she put her finger on her lateral epicondyle. Which of the following statements best describes the most likely cause of her elbow pain?

The answer is lateral epicondylitis of the elbow. Chronic irritation/inflammation of the common extensor origin of the forearm extensor muscles results in inflammation and microrupture of the soft tissues of the lateral epicondyle of the humerus. This injury is the most common painful condition of the elbow region. It is sometimes called tennis elbow. Calcification of tissues around the lateral epicondyle is incorrect. This condition could produce pain in the common extensor tendon, but it would unlikely be as severe as was experienced by the novice tennis player. It is also unlikely to develop over a 3-week period. The woman probably would have felt pain at the beginning of her tennis playing if it were a preexisting condition. Rupture of the common extensor tendon is incorrect. Chronic inflammation of this tendon could eventually result in rupture of this substantial structure; however, the injury experienced by this tennis player developed within 3 weeks. Also, such a rupture would result in very acute pain and functional deficits—not just pain. Avulsion of the lateral epicondyle of the elbow is incorrect. This injury could possibly result from repetitive extension of the forearm but it is much less common than lateral epicondylitis. The resulting traction of the tendon on the epicondyle could avulse (tear off) this projection from the lateral side of the distal end of the humerus. This would also result in apparent functional deficits. Bursitis in the subcutaneous olecranon bursa is incorrect and would not produce pinpoint pain on lateral epicondyle. (less)

Your young patient is interested in becoming the "chin-up" champion of his school. You instruct him that the muscle most important to develop to achieve his goal is which of the following?

The answer is latissimus dorsi. The latissimus dorsi is most important when raising the trunk to the arm, as in performing a chin-up.

An 18-year-old boy is cut severely on the lateral wall of his right chest during a knife fight. Following healing, his scapula moves away from the thoracic wall when he leans on his right hand, giving the appearance indicated in the given photo. Which of the following nerves is likely damaged?

The answer is long thoracic nerve. The photo demonstrates a case of "winged scapula," indicative of a lesion of the long thoracic nerve and subsequent paralysis of the serratus anterior muscle. The nerve runs down the lateral thoracic wall, on the superficial aspect of the serratus anterior, where it is unusually exposed (for a motor nerve) and vulnerable to injury, especially when the limb is elevated. Lesion of the nerve denervates the serratus anterior. This results in the medial border and inferior angle of the scapula pulling away from the posterior chest wall, giving the scapula a wing-like appearance when the affected limb is protracted. Additionally, the affected arm cannot be abducted above the horizontal plane because the serratus anterior is not available to superiorly rotate the glenoid cavity of the scapula to allow full abduction. Axillary nerve is incorrect. This nerve passes deeply through the axilla, around the surgical neck of the humerus, to supply the teres minor and deltoid muscles. Lesion here would result in significant weakness in abduction of the arm and wasting of the rounded contour of the shoulder. Thoracodorsal nerve is incorrect. This nerve runs inferior through the axilla to supply the latissimus dorsi muscle. Loss of the nerve would result in weakness in extension and medial rotation of the arm, plus wasting of the posterior axillary fold. Dorsal scapular nerve is incorrect. This nerve courses into the upper, medial part of the back and the lower neck to supply the levator scapulae and rhomboid muscles. Paralysis of these muscles would result in weakness in elevation and retraction of the scapula and perhaps wasting of the contour of the back under the trapezius muscle. Suprascapular nerve is incorrect. The suprascapular nerve runs through the suprascapular notch and into the supraspinous and infraspinous fossae to supply the supraspinatus and infraspinatus muscles. Lesion of this nerve would result in weakness in the rotator cuff affecting the initiation of abduction (supraspinatus) and external rotation (infraspinatus) of the shoulder and wasting of the muscular contour of the posterior aspect of the scapula. (less)

A 16-year-old boy fell from a motorcycle, and his radial nerve was severely damaged because of a fracture of the midshaft of the humerus. Which of the following conditions would most likely result from this accident?

The answer is loss of wrist extension leading to wrist drop. Injury to the radial nerve results in loss of wrist extension, leading to wrist drop. The median nerve innervates the pronator teres, pronator quadratus, and opponens pollicis muscles and the skin over the ventral aspect of the thumb. The ulnar nerve innervates the dorsal interosseous muscles, which act to abduct the fingers. (less)

A 19-year-old man arrives at his campus health clinic complaining of soreness in his right wrist. He explains he landed on an outstretched hand when he was tackled in a rugby match. He indicates that the pain worsens with movement and is minimized by stabilization of the wrist. There are no sensory deficits in his hand nor does he have trouble grasping or holding objects. Pressure applied to the anatomic snuffbox between the extensor pollicis brevis and extensor pollicis longus tendons produces no pain. Radiographic studies show no fractures but reveal an anterior dislocation of a bone in the proximal row of carpal bones. What carpal bone is most likely dislocated in this patient?

The answer is lunate. The lunate is shaped like a moon, thus its name. It is situated in the center of the proximal row of carpal bones where it articulates with the radius. This bone is the most commonly dislocated carpal bone, which leads to severe carpal instabilities. This dislocation often occurs in association with a trans-scaphoid fracture. It is important to note that scaphoid fractures are often difficult to see in radiographic imaging; however, these fractures can be detected frequently by applying direct pressure to the anatomical snuffbox. Scaphoid is incorrect. The scaphoid bone is located in the proximal row and is the most frequently fractured carpal bone. As the weight of the body is transmitted through the upper limb onto the outstretched hand and the impact of the fall exerts pressure back upon the limb, the scaphoid bone is crushed by these opposing forces and is subsequently fractured, usually along its narrowest part. The scaphoid bone is clinically relevant due to the frequency of fractures in younger patients and the poor vascularization of its proximal part. Avascular necrosis is often a postfracture complication that slows the healing of this bone. Capitate is incorrect. The capitate is located in the distal row of carpal bones, so this selection can be easily eliminated. Triquetrum is incorrect. The triquetrum is located in the proximal row of carpal bones and may be involved in a severe dislocation of the wrist. Specifically, the triquetrum can be displaced in Stage III wrist dislocations when the triquetrolunate interosseous ligament is damaged. However, tearing this interosseous intercarpal ligament would only occur following dislocation of the lunate bone, which is the most commonly dislocated carpal bone. Trapezium is incorrect. The trapezium bone helps form the distal floor of the anatomic snuffbox and forces would be exerted on this bone during the fall. However, it is not located in the proximal row of carpal bones. Moreover, this four-sided bone is rarely fractured or dislocated due to its shape and construction. (less)

Your 33-year-old patient has warm, tender red streaks in the skin of her upper limb and is very tender in her axilla. What do you suspect?

The answer is lymphangitis. Lymphangitis (inflammation of lymphatic vessels) in the upper limb is characterized by the development of warm and tender streaks in the skin.

You are about to insert a catheter into the vein of your 33-year-old patient that runs diagonally across the anterior surface of the elbow joint (cubital fossa). What is this vein?

The answer is median cubital. The median cubital vein traverses the cubital fossa, connecting the cephalic and basilic veins.

The secretary of a rather verbose academic physician in internal medicine complains of numbness and tingling in her hands and fingers. She is constantly typing long patient visit dictations and now has carpal tunnel syndrome, which is due to compression of which of the following structures?

The answer is median nerve. In carpal tunnel syndrome, structures entering the palm deep to the flexor retinaculum are compressed; these include the median nerve and the tendons of the flexor pollicis longus, flexor digitorum profundus, and flexor digitorum superficialis muscles. The flexor carpi radialis runs lateral to the carpal tunnel and inserts on the bases of the second and third metacarpals. Structures entering the palm superficial to the flexor retinaculum include the ulnar nerve, ulnar artery, and palmaris longus tendon (which inserts on the palmar aponeurosis). (less)

A 35-year-old man walks in with a stab wound to the most medial side of the proximal portion of the cubital fossa. Which of the following structures would most likely be damaged?

The answer is median nerve. The contents of the cubital fossa from medial to lateral side are the median nerve, the brachial artery, the biceps brachii tendon, and the radial nerve. Thus, the median nerve is damaged. The radial recurrent artery ascends medial to the radial nerve. (less)

Physical examination of a 40-year-old man injured in an automobile accident indicates that he has suffered nerve damage affecting his left upper limb. The patient exhibits significant weakness when pronating his left forearm and flexing his left wrist. What nerve is most likely damaged?

The answer is median nerve. The median nerve controls pronation through the actions of the pronator teres and pronator quadratus muscles in the anterior compartment of the forearm. It also controls much of flexion of the wrist and lateral digits via the actions of most of the other muscles in that compartment. Ulnar nerve is incorrect. The ulnar nerve controls 1½ muscles in the anterior compartment of the forearm (flexor carpi ulnaris and the ulnar half of the flexor digitorum profundus) and most of the intrinsic muscles of the hand. However, neither of these 1½ forearm muscles produces pronation. Superficial branch of the radial nerve is incorrect. The superficial branch of the radial nerve is entirely cutaneous, carrying sensation from the dorsolateral part of the hand. So, cutting this nerve would not result in weakness in pronation of the forearm or flexion of the wrist. Deep branch of the radial nerve is incorrect. The deep branch of the radial nerve supplies the posterior compartment of the forearm. It influences supination via motor control of the supinator muscle, but not pronation. Musculocutaneous nerve is incorrect. The musculocutaneous nerve supplies the anterior compartment of the arm, and then continues distally via its termination as the lateral cutaneous nerve of the forearm. It contributes significantly to control of supination by its innervation of the biceps brachii muscle, but has no effect on wrist flexion. (less)

A child is born to a young woman who had utilized thalidomide to help relieve her morning sickness early in her pregnancy. The infant suffers the congenital defects shown here. She is missing the proximal segments of both upper and lower limbs. The hands and feet that are present are attached to the trunk of the body and resemble small seal's flippers. Which of the following is the correct term for this malformation?

The answer is meromelia. Thalidomide was once used widely to help combat morning sickness during pregnancy. Following a wave of children born with limb malformations, the medication was discovered to be a significant teratogen and was taken off the market. Currently, thalidomide is being used to treat AIDS and cancer patients, raising concerns of a new rash of children born with limb defects. The specific malformation in this case is termed meromelia, a partial absence of limb segments. Meromelia is a type of reduction defect, in which whole or partial limb components are absent. Because of the resemblance of the limbs to a seal's flippers, this defect is also termed phocomelia ("seal limb"). A dysplasia is incorrect. Dysplasias are malformations in which elements are malformed. In this case, limb components are missing rather than malformed. A duplication defect is incorrect. Duplication defects are characterized by the presence of supernumerary (extra) elements. An example is polydactyly, in which extra digits are present. Micromelia is incorrect. Micromelia ("small limb") is a type of dysplasia. In this, the limb segments are present but are abnormally short. Amelia is incorrect. Amelia ("without limb") is a type of reduction defect in which an entire extremity is absent. (less)

A physician tests the myotatic biceps reflex as shown. A normal response of involuntary contraction of the biceps brachii muscle is noted. This reflex confirms the integrity of what nerve?

The answer is muculocutaneous nerve. A positive response to the myotatic biceps reflex confirms the integrity of the musculocutaneous nerve and the C5 and C6 spinal segments, from which this nerve arises. The musculocutaneous nerve supplies motor innervation and proprioception to the muscles in the anterior compartment of the arm, including the coracobrachialis, biceps brachii (tested here), and brachialis. Lesioning the musculocutaneous nerve would lead to loss of proprioception and weakness in flexing the elbow (via the biceps brachii and brachialis muscles) and supinating the forearm (via the biceps brachii), resulting in a negative myotatic biceps reflex. Axillary nerve is incorrect. The axillary nerve innervates the deltoid and teres minor muscles as well as the skin overlying the deltoid in the superolateral aspect of the arm. Loss of innervation to the deltoid muscle would cause weakness in abduction of the upper limb. The teres minor and posterior head of the deltoid are responsible for external rotation of the shoulder. Median nerve is incorrect. The median nerve innervates 6.5 of the 8 anterior forearm muscles. This nerve would help with flexion of the forearm at the elbow; however, the myotatic biceps reflex test specifically tests the innervation of the biceps brachii muscle, which is innervated by the musculocutaneous nerve. Ulnar nerve is incorrect. The ulnar nerve innervates only 1.5 muscles of the forearm, specifically the flexor carpi ulnaris and the ulnar side of the flexor digitorum profundus. This nerve also innervates most of the intrinsic muscles of the hand. It is not involved with the myotatic biceps reflex test. Radial nerve is incorrect. The radial nerve supplies motor innervation to the posterior compartments of the arm and forearm. Damage to this nerve would cause weakness in extension at the elbow and wrist joints. The radial nerve is responsible for the posterior cutaneous nerves of the arm and forearm as well as the inferior lateral cutaneous nerve of the arm. However, it is responsible for extension at the elbow, not the flexion of the elbow tested with the myotatic biceps reflex test. (less)

A patient with Bennett's fracture (a fracture of the base of the first metacarpal bone) experiences an impaired thumb movement. Which of the following intrinsic muscles of the thumb is most likely injured?

The answer is opponens pollicis. The opponens pollicis inserts on the first metacarpal. All other intrinsic muscles of the thumb, including the abductor pollicis brevis, the flexor pollicis brevis, and the adductor pollicis muscles, insert on the proximal phalanges. (less)

The ulnar nerve innervates _______________.

The answer is part of flexor digitorum profundus. Flexor digitorum profundus has a dual innervation. The ulnar (medial) half is innervated by the ulnar nerve, while the radial (lateral) half is innervated by the median nerve. (less)

A 20-year-old man fell from the parallel bar during the Olympic trial. A neurologic examination reveals that he has a lesion of the lateral cord of the brachial plexus. Which of the following muscles is most likely weakened by this injury?

The answer is pectoralis major. The pectoralis major is innervated by the lateral and medial pectoral nerves originating from the lateral and medial cords of the brachial plexus, respectively. The subscapularis, teres major, latissimus dorsi, and teres minor muscles are innervated by nerves originating from the posterior cord of the brachial plexus. (less)

Organization of the axillary lymph nodes into Levels I, II, and III for breast cancer treatment is based on the location of the nodes relative to which of the following muscles?

The answer is pectoralis minor. The pectoralis minor muscle forms a portion of the anterior axillary wall and is used as a marker to organize the contained groups of axillary lymph nodes. In a clinical context, the axillary nodes are organized into three major groups (Level 1, 2, and 3) that denote the progressive lymph drainage of the breast. Level 1 (anterior; pectoral) nodes are located along the lateral border of the pectoralis minor. Level 2 (central; deep) nodes are situated in the center of the axilla, deep (posterior) to the pectoralis minor. Level 3 (apical; medial) nodes are positioned in the apex of the axilla, superior to the upper medial border of the pectoralis minor. Detection of cancer cells in each increasing number level of nodes indicates progressively greater metastasis of the disease out of the breast. Pectoralis major is incorrect. The pectoralis major and minor form the anterior axillary wall. However, the pectoralis major muscle is not used to classify the positions of the axillary lymph nodes or levels of metastasis. Latissimus dorsi is incorrect. The latissimus dorsi muscle helps to form the posterior wall of the axilla along with the subscapularis and teres major muscles. Due to its posterior location, it is not an important landmark for classification of axillary lymph nodes in breast cancer. Serratus anterior is incorrect. This muscle contributes to the formation of the medial wall of the axilla, along with the thoracic cage. This muscle is not an important landmark for classification of axillary lymph nodes in breast cancer. Subscapularis is incorrect. The subscapularis muscle helps to form the posterior wall of the axilla along with the latissimus dorsi and teres major muscles. Due to its posterior location, this muscle is not an important landmark for classification of axillary lymph nodes in breast cancer. (less)

The given X-ray reveals a fracture of the proximal humerus, indicated by the black arrow. Given the location of the fracture, what artery is most likely damaged in this patient?

The answer is posterior circumflex humeral artery. The radiograph shows a fracture of the surgical neck of the humerus. The posterior humeral circumflex artery, accompanied by the axillary nerve, lies against the posterior aspect of the surgical neck as it passes into the quadrangular space of the shoulder. This fracture places both of these structures in immediate danger. Brachial artery is incorrect. The brachial artery travels down the midline of the arm, close to the median nerve. It is not immediately endangered by the fracture of the surgical neck. Deep brachial, or profunda brachii, artery is incorrect. The deep brachial artery wraps tightly around the midshaft of the humerus, in the radial (spiral) groove. It, along with the radial nerve with which it travels, would be endangered by a fracture through the radial groove of the humerus. Subscapular artery is incorrect. The subscapular artery is the largest branch of the axillary artery. It descends along the axillary border of the scapula and is not in contact with the humerus. Superior ulnar collateral artery is incorrect. This artery is a branch of the brachial artery. It descends through the arm, moves into company with the ulnar nerve, and takes a close relation to the posterior aspect of the medial epicondyle of the humerus. This vessel would be endangered by a fracture of the medial epicondyle rather than a surgical neck fracture. (less)

A 10-year-old boy falls off his bike, has difficulty in moving his shoulder, and is brought to an emergency department. His radiogram and angiogram reveal fracture of the surgical neck of his humerus and bleeding from the point of the fracture. This accident most likely leads to damage of which of the following arteries?

The answer is posterior humeral circumflex. The posterior humeral circumflex artery accompanies the axillary nerve that passes around the surgical neck of the humerus. None of the other arteries are involved. (less)

The given anteroposterior (AP) X-ray depicts a humeral shaft fracture in a 22-year-old man. Given the location of the fracture, which of the following structures is most likely damaged?

The answer is profunda brachii artery. It is important to recognize where neurovascular structures have close positional relations to each other and to underlying bony structures in order to predict the likely second order functional consequences of damage to the bones. In the given AP X-ray, the midshaft of the humerus is fractured slightly distal to the radial groove. At this point, the profunda brachii vessels (deep vessels of the arm) and the radial nerve emerge from the radial groove in a bundle tightly wrapped against the body of the humerus. A fracture here may readily damage any of these neurovascular structures. Lesion of the vessels may produce swelling in the posterior compartment of the arm and loss of supply to the muscles therein. Lesion of the nerve will result in major motor and sensory deficits in the posterior aspect of the forearm and hand. Posterior circumflex humeral artery is incorrect. This vessel travels in companionship with the axillary nerve around the surgical neck of the humerus. Fractures here are the most common injuries to the proximal end of the humerus, especially in the elderly. Ulnar nerve is incorrect. In the arm, the ulnar nerve travels with the superior ulnar collateral artery. Both lie in contact with the posterior side of the large, projecting medial epicondyle of the humerus as they cross the elbow. Axillary nerve is incorrect. The axillary nerve travels in companionship with the posterior circumflex humeral artery around the surgical neck of the humerus. Fractures here are the most common injuries to the proximal end of the humerus, especially in the elderly. Median nerve is incorrect. This large nerve travels with the brachial vessels down the anterior midline of the arm. The nerve lies close to the distal end of the humerus, where it may be damaged due to fractures of the condyle. (less)

A 38-year-old homebuilder was involved in an accident and is unable to supinate his forearm. Which of the following nerves are most likely damaged?

The answer is radial and musculocutaneous. The supinator and biceps brachii muscles, which are innervated by the radial and musculocutaneous nerves, respectively, produce supination of the forearm. This is a question of two muscles that can supinate the forearm. (less)

A 14-year-old boy falls on his outstretched hand and has a fracture of the scaphoid bone. The fracture is most likely accompanied by a rupture of which of the following arteries?

The answer is radial artery. The scaphoid bone forms the floor of the anatomic snuffbox, through which the radial artery passes to enter the palm. The radial artery divides into the princeps pollicis artery and the deep palmar arch. (less)

A 21-year-old male college student reports to the student health clinic on Monday morning, the day after the Super Bowl. He explains that he was intoxicated and lost consciousness with his upper limbs draped over the back of a couch. He complains of numbness and paresthesia over the dorsum of his hand on the radial side and is unable to support the weight of his left hand when the hand is placed in a pronated position (see photo). What nerve was most likely damaged in this individual?

The answer is radial nerve. This case represents a classic presentation of "Saturday Night Palsy," where the radial nerve is compressed against the humerus in the arm. Remember, the radial nerve supplies motor innervation to the posterior compartments of the arm and forearm, so damage to this nerve would cause weakness in extending the elbow and wrist. This patient is unable to extend the wrist when the hand is placed in a pronated position ("wrist drop"), implying damage to the radial nerve. Moreover, the superficial branch of the radial nerve is responsible for cutaneous innervation over much of the dorsum of the hand, which explains the numbness and paresthesia in his hand. Axillary nerve is incorrect. The axillary nerve innervates the deltoid and teres minor muscles as well as the skin overlying the deltoid in the superolateral aspect of the arm. Compressing the axillary nerve would not affect wrist function or cause paresthesia distal to the wrist, which was seen in this student. Median nerve is incorrect. The median nerve is responsible for flexion at the wrist; however, this patient is having trouble with wrist extension. The sensory distribution for the median nerve is primarily on the palmar side of the hand rather than the dorsum of the hand, which is seen in this patient. Ulnar nerve is incorrect. The ulnar nerve is responsible for sensory innervation on the dorsum of the hand that is limited to the ulnar side (medial to the midline of the fourth finger). Damage to the ulnar nerve would not result in "wrist drop," which is an indication of a radial nerve injury. The ulnar nerve does supply motor innervation to most of the intrinsic muscles of the hand. Musculocutaneous nerve is incorrect. The musculocutaneous nerve supplies motor innervation to the anterior compartment of the arm and cutaneous innervation to the lateral aspect of the forearm. Therefore, damage to this nerve would not affect wrist function. (less)

You are seeing a patient in the ED after a skiing accident in which she sustained a mid-humeral shaft fracture. She is displaying a wrist-drop. You suspect injury to which of the following nerves?

The answer is radial. Wrist drop is a classic sign of radial nerve damage. It results because the extensor muscles of the wrist and hand are paralyzed due to the rupture of the radial nerve as it traverses around the humeral shaft. (less)

A 10-year-old boy was running across a parking lot when he tripped and received lacerations on the base of his thumb from a broken glass bottle. On examination, his thumb was unable to oppose to his fingers, and the thumb also showed weakness when abducting and flexing. No sensory deficits were reported. What nerve was most likely severed?

The answer is recurrent branch of median nerve. The recurrent (thenar) branch of the median nerve lies subcutaneously in the thenar eminence and can be damaged by lacerations in this area. This nerve innervates most of the thenar muscles, including the Opponens pollicis, Abductor pollicis brevis, and superficial head of the Flexor pollicis brevis (mnemonic = "OAF"). Opposition of the thumb would be lost by cutting the recurrent branch of the median nerve. Though the abductor pollicis brevis is denervated, the abductor pollicis longus, innervated by the deep branch of the radial nerve, is still intact. Also, the flexor pollicis longus, innervated by the anterior interosseous nerve, would still allow flexion at the interphalangeal joint of the thumb. Cutting the recurrent branch of the median nerve would lead to atrophy of the thenar muscular complex, a condition known as "ape hand." Deep branch of the radial nerve is incorrect. The deep branch of the radial nerve is also called the posterior interosseous nerve as it exits from the supinator muscle. It innervates the abductor pollicis longus and extensor pollicis longus and brevis muscles, which form the boundaries of the anatomical snuffbox. Cutting this nerve would cause loss of extension of the thumb and weakness in abduction. However, the puncture was on the thenar eminence, and the radial nerve does not travel there. Superficial branch of the radial nerve is incorrect. The superficial branch of the radial nerve arises from the radial nerve in the cubital fossa. This nerve is purely cutaneous, supplying sensation to the dorsum of the hand and fingers. Because it does not have a motor component, this nerve would not be responsible for the deficit in thumb function. Deep branch of ulnar nerve is incorrect. The deep branch of the ulnar nerve innervates most of the intrinsic muscles of the hand, including the hypothenar muscles, medial two lumbrical muscles, the palmar and dorsal interossei, adductor pollicis, and the deep head of the flexor pollicis brevis. With a small motor supply to the thenar compartment (specifically the deep head of the flexor pollicis brevis), this nerve could not cause a substantial loss of function in the thenar eminence. Superficial branch of the ulnar nerve is incorrect. The superficial branch of the ulnar nerve arises from the ulnar nerve distal to the flexor retinaculum. This nerve supplies cutaneous branches to the anterior surface of the medial one and a half fingers. The palmaris brevis is the only muscle supplied by this nerve, and this muscle tightens the skin of the medial surface of the palm. Therefore, this nerve would not be responsible for substantial loss of motor function in the thumb. (less)

A 33-year-old tennis player initially experienced vague pain in the anterior region of her glenohumeral joint during a game. Then, she had the sensation of something popping during her third set. During the preliminary examination by her trainer, he noted tenderness over the intertubercular groove in the humerus, flexion and supination weakness of the forearm, and an abnormal bulge in the distal part of the anterior part of her arm. Radiographs of the shoulder and arm did not reveal skeletal abnormalities. Which of the following statements best describes the most likely cause of the symptoms and signs in this case?

The answer is rupture of the tendon of the long head of the biceps. The popping sensation and tenderness over the intertubercular groove of the humerus suggest that the tendon of the long head of the biceps was ruptured. The bulge in the arm resulted from the release of the belly of the biceps when the tendon ruptured. Avulsion of the supraglenoid tubercle of the scapula is incorrect. If this avulsion occurred, a fracture would have been apparent in the radiographs. No fracture was detected. Rupture of the tendon of the short head of the biceps is incorrect. The proximal attachment of this head is to the tip of the coracoid process of the scapula. The short head does not have a slender tendon that is as likely to rupture in the same manner as the long head. Further rupture of the short head tendon would not produce tenderness over the intertubercular groove. Rupture of the long head of the triceps brachii tendon is incorrect. This tendon does not traverse the intertubercular groove. Rupture of the subscapularis tendon is incorrect. This tendon does not traverse the intertubercular groove. (less)

A 21-year-old man goes to his college campus health clinic complaining of soreness in his left wrist after falling on an outstretched hand during a basketball game the previous day. He is supporting his left wrist and indicates that the pain worsens with movement and is minimized with inactivity. There is no loss of feeling in his hand, nor does he have trouble grasping or holding objects. The physician exacerbates the wrist pain by applying pressure to the base of the thumb in the anatomical snuffbox (see photo). Radiographic imaging will confirm a break of which carpal bone?

The answer is scaphoid. The most commonly fractured carpal bone is the scaphoid that forms the floor of the anatomical snuffbox. This area is a fossa located between the three long tendons of the thumb (tendons of abductor pollicis longus and extensor pollicis brevis laterally and tendon of the extensor pollicis longus medially). The scaphoid bone is frequently broken when an individual falls with an outstretched hand and lands on the palm with the hand abducted. A broken scaphoid bone is commonly seen in individuals under the age of 30. The scaphoid is broken due to its unfortunate position between the downward force transmitted by the weight of the upper limb and the upward force due to the impact of hitting the floor. The proximal aspect of a broken scaphoid bone can suffer from avascular necrosis due to its blood supply entering the bone distally. Capitate is incorrect. The capitate is located centrally in the distal row of carpal bones and articulates with most of the carpal bones (the triquetrum, pisiform, and trapezium being the exceptions). This bone is not located in the floor of the anatomical snuffbox, so it was not the most likely bone injured in this patient. Hamate is incorrect. The hamate is located in the distal row of carpal bones on the ulnar side. It is not related to the anatomical snuffbox, so it would not produce pain in the base of the thumb when broken. Trapezium is incorrect. The trapezium is located at the base of the thumb, but it is not commonly fractured when a person falls with an outstretched hand. A mnemonic for the position of the trapezium is "trapezium articulated with the thumb." Pisiform is incorrect. The pisiform is located in the proximal carpal row and is a sesamoid bone located within the tendon of the flexor carpi ulnaris muscle. Its position on the ulnar side of the wrist would not cause pain in the anatomical snuffbox. (less)

You are examining the hand of a patient after a fall in which she landed on her outstretched hand. When you palpate deeply in the floor of the anatomical snuff box, you are most likely feeling which of the following?

The answer is scaphoid. The scaphoid can be palpated deep in the floor of the anatomical snuff box.

A 24-year-old wrestler is complaining to you of pain and swelling in his elbow overlying his olecranon. He denies any recent serious trauma. What do you initially suspect?

The answer is subcutaneous olecranon bursitis. This condition is common in wrestlers due to repeated pressure and friction on the olecranon.

In viewing an upper limb angiogram of your 76-year-old patient, you are surprised to see that all but the most distal part of her right axillary artery is totally occluded yet her entire upper limb remains healthy. You suspect blood is reaching her upper limb via an anastomosis between which of the following?

The answer is subscapular and dorsal scapular. The subscapular artery anastomoses with the suprascapular, dorsal scapular and intercostal arteries.

Your 48-year-old patient is having some difficulty extending her wrist and you believe she is suffering from a compression syndrome in which a muscle is compressing the deep branch of her radial nerve. Because elbow extension is normal, you surmise that the muscle most likely to be exerting the compression is which of the following?

The answer is supinator. The deep branch of the radial nerve passes between the superficial and deep parts of the muscle and is therefore susceptible to compression from this muscle. (less)

You are treating a patient in the ED with a fracture of the proximal end of the humerus. Most of these fractures occur at which of the following?

The answer is surgical neck. The surgical neck is that part of the humerus that is distal to the head and tubercles and is the most frequent site for a proximal humeral fracture. (less)

A patient has a torn rotator cuff of the shoulder joint as the result of an automobile accident. Which of the following muscle tendons is intact and has normal function?

The answer is teres major. The rotator cuff consists of the tendons of the supraspinatus, infraspinatus, subscapularis, and teres minor muscles. It stabilizes the shoulder joint by holding the head of the humerus in the glenoid cavity during movement. The teres major inserts on the medial lip of the intertubercular groove of the humerus. (less)

A 10-year-old boy falls off his bike, has difficulty in moving his shoulder, and is brought to an emergency department. His radiogram and angiogram reveal fracture of the surgical neck of his humerus and bleeding from the point of the fracture. Following this accident, the boy has weakness in rotating his arm laterally. Which of the following muscles are paralyzed?

The answer is teres minor and deltoid. The lateral rotators of the arm include the teres minor, deltoid, and infraspinatus muscles, but the infraspinatus muscle is innervated by the suprascapular nerve. (less)

You suspect the patient you are examining in the ED has a minor radial nerve injury that primarily has affected the superficial branch of this nerve. To test the function of this superficial branch, you would do which of the following?

The answer is test for sensation on the dorsal aspect of the proximal lateral hand. The superficial branch of the radial nerve carries only sensory fibers from the dorsal aspect of the proximal lateral hand. (less)

Your patient has an infection of the tendon and synovial sheaths of his index finger. Which of the following is most likely?

The answer is the infection will stay confined to the sheath of this finger. Because the tendon of the 2nd finger usually has a separate synovial sheath, infection is usually confined to this finger whereas infection of the fifth finger can easily spread into the forearm. (less)

The musculocutaneous nerve continues into the forearm as the _______________.

The answer is the lateral cutaneous nerve of the forearm. As the musculocutaneous nerve emerges from deep to the biceps brachii in the distal arm, it becomes cutaneous, and the name changes to the lateral cutaneous nerve of the forearm. (less)

A 55-year-old professional piano player visits his physician with a complaint of gradually worsening pain and weakness in his left hand. The pain consists of a burning sensation in the lateral three and a half fingers, but not in the thenar eminence. The thenar muscles are noticeably weak, which is particularly obvious during opposition of the thumb. There are no apparent deficits proximal to the wrist. This condition is most likely the result of compression of which of the following nerves?

The answer is the median nerve. The described condition is a classic example of carpal tunnel syndrome produced by compression of the median nerve within the carpal tunnel. The median nerve supplies the three thenar muscles, the first two lumbricals, and the skin of the palmar side of the lateral 3.5 digits, including their nail beds. Restriction of the deficits to this area indicates the median nerve is compressed at the wrist rather than more proximally. Deep branch of the radial nerve is incorrect. The deep branch of the radial nerve supplies the muscles of the posterior compartment of the forearm and the carpal joints. It does not usually enter the hand. Musculocutaneous nerve is incorrect. The musculocutaneous nerve supplies the muscles in the anterior compartment of the arm and the skin along the lateral side of the forearm. It does not normally enter the hand. Superficial branch of the radial nerve is incorrect. The superficial branch of the radial nerve is entirely sensory. It crosses the posterolateral side of the wrist and supplies the posterolateral aspect of the hand and the posterior proximal parts of the lateral 3.5 digits. Ulnar nerve is incorrect. The ulnar nerve has an extensive distribution in the hand. However, its cutaneous territory is the medial aspect of the palmar and dorsal sides of the hand and the medial 1.5 digits. The ulnar nerve also supplies all the intrinsic muscles of the hand except for the five controlled by the median nerve. (less)

A 36-year-old plumber slips and breaks a porcelain sink, cutting an anterior aspect of his wrist deeply on a sharp edge. On arrival to the emergency department, he can adduct his thumb but not oppose it. Which of the following structures has been severed?

The answer is the median nerve. The median nerve enters the palm of the hand through the carpal tunnel deep to the flexor retinaculum, giving off a muscular branch (recurrent branch) to the thenar muscles including the abductor pollicis brevis, flexor pollicis brevis, and opponens pollicis. The ulnar nerve enters the hand superficial to the flexor retinaculum and lateral to the pisiform bone, supplying the hypothenar muscles and adductor pollicis. The patient can adduct his thumb but not oppose it. Therefore, the median nerve is injured. The radial nerve and the anterior and posterior interosseous nerves do not supply the muscle that opposes the thumb. (less)

A 42-year-old woman presents to an outpatient clinic with a 6-month history of numbness and tingling on the palmar aspect of her lateral three and one-half fingers, loss of pronation, and flattening of the thenar eminence. Injury to which of the following nerves could cause such a condition?

The answer is the median nerve. The median nerve supplies the skin on the palmar aspect of the lateral three and one-half fingers and the dorsal side of the index finger, middle finger, and one-half of the ring finger. The median nerve innervates the pronator teres and pronator quadratus muscles and the thenar muscles. The axillary and musculocutaneous nerves do not supply the skin or muscles of the hand. The radial nerve does not innervate muscles of the hand but innervates the skin of the radial side of the hand and the radial two and one-half digits over the proximal phalanx. The ulnar nerve innervates not only the palmaris brevis, hypothenar muscles, adductor pollicis, dorsal and palmar interosseous, and medial two lumbrical muscles but also the skin over the palmar and dorsal surfaces of the medial one-third of the hand and the skin of the little finger and the medial side of the ring finger. (less)

A 17-year-old boy is taken to the emergency room after being struck on the lateral aspect of the left shoulder by a baseball bat during a gang fight. Examination reveals a fracture of the surgical neck of the left humerus, displaced bone fragments piercing the quadrangular space of the shoulder, and deep blood accumulation around the fracture. Which of the following blood vessels was most likely severed?

The answer is the posterior circumflex humeral artery. This vessel, along with the axillary nerve, originates in the axilla and passes through the quadrangular space to reach the posterior aspect of the shoulder. This neurovascular bundle is in immediate danger in cases of trauma or other conditions (e.g., muscle hypertrophy) that affect the integrity of the quadrangular space. Cephalic vein is incorrect. The cephalic vein is a subcutaneous vessel that passes across the anterior aspect of the shoulder. It is far removed from the quadrangular space. Circumflex scapular artery is incorrect. The circumflex scapular artery is a branch of the subscapular artery. It passes around the axillary border of the scapula and through the triangular space of the shoulder to contribute to the arterial anastomoses around the shoulder joint. Profunda brachii artery is incorrect. The profunda brachii vessels and the accompanying radial nerve are closely related to the midshaft of the humerus, where they may be damaged as a result of fracture. Subscapular artery is incorrect. The subscapular artery runs downward along the axillary border of the scapula. It courses away from the quadrangular space. (less)

The most frequently fractured carpal bone is the ____________.

The answer is the scaphoid. Falls on an outstretched hand often fracture the scaphoid. The scaphoid forms the floor of the anatomical snuffbox, which contains the radial artery. Fracture of this bone may result in damage to the artery. (less)

A 22-year-old woman professional soccer player falls hard onto her outstretched right hand during a game, injuring her wrist. Manipulation of the wrist during postgame physical examination by the team physician elicits deep pain in the anatomical snuffbox. Follow-up radiographs will most likely reveal a fracture of which of the following bones?

The answer is the scaphoid. The scaphoid is the most lateral (radial side; thumb side) element in the proximal row of carpal bones, and can be palpated in the floor of the anatomical snuff box. The scaphoid is the most commonly fractured carpal bone. Deep tenderness and/or pain in the snuffbox following an injury of the type described here are strong indicators of trauma to the scaphoid. Base of the fifth metacarpal is incorrect. The base of the fifth metacarpal bone is located in the medial (ulnar) aspect of the wrist, whereas the anatomical snuff box is in the lateral (radial) side of the wrist. Hamate is incorrect. The hamate bone is the most medial (ulnar) element in the distal row of carpal bones. It is well removed from the anatomical snuff box. Head of the radius is incorrect. The head of the radius is located at the proximal (elbow) end of the radius. However, the small head of the ulna is situated at the distal (wrist) end of that bone. Pisiform is incorrect. The pisiform bone is the most medial (ulnar) of the four elements forming the proximal row of carpal bones. It is a sesamoid bone within the tendon of the flexor carpi ulnaris muscle, and is not related to the anatomical snuff box. (less)

A 35-year-old left-handed baseball pitcher experiences a severe spasm of pain while abducting his left arm in beginning to throw a pitch, and is unable to continue playing. Subsequently, he cannot initiate abduction of the left arm. However, if that arm is passively elevated through the first 15° of abduction, he can complete bringing the arm up to a right angle. The tendon of which of the following muscles is most likely torn?

The answer is the supraspinatus. The supraspinatus, one of the four rotator cuff muscles, has the primary action of initiating abduction of the arm. It is the only rotator cuff member that does not have a main action of rotating the arm. The supraspinatus tendon is subject to possible degenerative changes related to subacromial bursitis or other overuse syndromes, which may lead to its rupture. In case of tendon rupture, the patient typically presents with the abduction motion deficit condition described here. Infraspinatus is incorrect. The infraspinatus is another of the rotator cuff muscles. However, its primary action is to laterally rotate the arm. Trauma to this muscle would not result in the type of abduction deficit described in this case. Subscapularis is incorrect. The subscapularis, another rotator cuff muscle, acts mainly to medially rotate the arm. Teres major is incorrect. The teres major is not a rotator cuff muscle. However, it is located close by the rotator cuff and may be easily mistaken for part of that functional group. Its primary actions are to medially rotate and adduct the arm. Teres minor is incorrect. The teres minor is the fourth rotator cuff muscle. Its major action is to laterally rotate the arm. (less)

A rural physician making her first rounds in a medically isolated area meets a 45-year-old man with a postural deformity of his right hand similar to that shown in the figure. He explains the condition set in gradually over a period of several months after he "broke his elbow" falling off his roof. The physician immediately recognizes which of the following nerves was most likely injured?

The answer is the ulnar nerve. The condition shown here is an example of long-standing injury to the ulnar nerve at the elbow producing the characteristic "claw hand" (main en griffe) deformity. Muscle wasting and contracture are most pronounced in the third and fourth digits and across the body of the hand, but do not include the thenar eminence. The ulnar nerve is commonly injured at the elbow and often associated with trauma to the medial epicondyle of the humerus. Axillary nerve is incorrect. The axillary nerve supplies the deltoid and teres minor muscles. Long-term injury is expressed clinically by wasting of the deltoid and loss of the rounded contour of the shoulder. Median nerve is incorrect. The median nerve supplies most of the muscles in the anterior compartment of the forearm and several small muscles in the hand, including those of the thenar eminence. The notable clinical presentation of median nerve injury is "ape hand," which appears as wasting of the thenar eminence and a flattened, ape-like hand. Musculocutaneous nerve is incorrect. Injury to the musculocutaneous nerve is not characterized by any particular postural deformity. The muscles of the anterior compartment of the arm become wasted (except for part of the brachialis), and flexion of the shoulder and elbow are weakened. Radial nerve is incorrect. Long-standing injury to the radial nerve is often expressed as "wristdrop," in which the hand cannot be extended against gravity. However, the form of the hand itself is normal because the radial nerve does not supply any intrinsic hand muscles, and no muscle wasting occurs within the hand. (less)

A 31-year-old professional baseball pitcher suffers a torn medial collateral ligament of the elbow in his throwing arm. Two weeks later, he undergoes "Tommy John surgery," in which the tendon of the palmaris longus muscle is transplanted into the elbow to reconstruct the damaged ligament. During the operative procedure, the surgeon must be keenly aware of the close relationship of which of the following structures to the medial collateral ligament?

The answer is the ulnar nerve. The ulnar nerve crosses the elbow by passing behind the medial epicondyle of the humerus in close apposition to the medial collateral ligament (ulnar collateral ligament; UCL). The nerve must be treated with care during any reconstructive procedure at the medial side of the elbow. It may be involved in postoperative scar tissue formation, especially if transposed during the procedure. Brachial artery is incorrect. The brachial artery crosses the anterior aspect of the elbow. The vessel lies on the surface of the brachialis muscle and terminates at approximately the neck of the radius. Cephalic vein is incorrect. The cephalic vein is a subcutaneous vessel that crosses the elbow along the anterolateral side of the biceps brachii. It is far removed from the medial collateral ligament. Radial nerve is incorrect. The radial nerve winds around the back of the arm in the spiral (radial) groove of the humerus. The nerve pierces the lateral intermuscular septum and crosses the anterolateral aspect of the elbow between the brachialis and brachioradialis muscles. Ulnar artery is incorrect. The ulnar artery originates in the cubital fossa, anterior to the elbow, at the level of approximately the neck of the radius. It is one of the terminal branches of the brachial artery, and at its origin lies on the front of the tendon of the biceps brachii. (less)

A 57-year-old woman presents with increasing numbness of the fourth and fifth digits on her right hand. On examination, it is noticed that she has a wasted hypothenar eminence, inability to abduct the thumb, and a characteristic claw hand. Which of the following nerves has most likely been injured?

The answer is the ulnar nerve. The ulnar nerve supplies sensation to the fourth and fifth digits and innervates the hypothenar muscles, the dorsal interosseous muscles that abduct the fingers, and the medial half of the flexor digitorum profundus (to the ring and little fingers or fourth and fifth digits) that flexes the distal interphalangeal joints. Claw hand is a condition in which the ring and little fingers are hyperextended at the metacarpophalangeal joints and flexed at the interphalangeal joints. The axillary and anterior interosseous nerves do not supply the hand. Injury to the radial nerve results in wrist drop. Injury to the median nerve causes an ape hand (flattening of the thenar eminence). (less)

There are only 30 minutes left before the concert starts. The pianist, who incidentally has a cut over the palmar surface of her wrist, notices that she is unable to pick up a piece of music between her index and middle fingers. Which of the following nerves is most likely damaged?

The answer is the ulnar nerve. This pianist is unable to pick up a music piece between the index and middle fingers because she cannot adduct her index finger and abduct the middle finger. The adductor of the index finger is the palmar interosseous muscle, which is innervated by the ulnar nerve. Abductors of the middle finger are dorsal interosseous muscles, which are innervated by the ulnar nerve. The other nerves do not innervate adductors or abductors of the fingers. (less)

You are evaluating the scapulohumeral rhythm of your patient who has reduced shoulder movements. You know that for every 2 degree of elevation of the upper limb at the glenohumeral joint, which of the following is true?

The answer is there is one degree of movement at the scapulothorac joint. The scapulohumeral rhythm refers to the combined movements that occur at the glenohumeral joint and the scapulothoracic joint in a 2:1 ratio. (less)

Following a radical mastectomy procedure, a surgeon plans to conduct a breast reconstruction utilizing a latissimus dorsi muscle flap. What nerve will the surgeon need to keep intact during the surgical dissection of the chest wall to prevent atrophy of the muscle flap?

The answer is thoracodorsal nerve. Successful transposition of muscle flaps in reconstructive surgery hinges largely on maintaining the neurovascular pedicles that supply the muscle. The latissimus dorsi muscle is innervated by the thoracodorsal (middle subscapular) nerve, a branch of the posterior cord of the brachial plexus. Lesion of this nerve during dissection of the chest wall will cause loss of the muscle flap. Long thoracic nerve is incorrect. The long thoracic nerve supplies the serratus anterior muscle. Damage to this nerve would cause a "winged scapula." Intercostobrachial nerve is incorrect. The intercostobrachial nerve is actually the lateral cutaneous branch of the second thoracic nerve and is entirely sensory in innervation. This nerve arises from the chest wall underneath the second rib and travels distally to give sensory innervation to the skin of the medial arm. Medial pectoral nerve is incorrect. The medial pectoral nerve innervates the pectoralis minor and major muscles. It would have been cut during the removal of these muscles during the radical mastectomy. Axillary nerve is incorrect. The axillary nerve traverses the quadrangular space (accompanying the posterior circumflex humeral artery) to supply the teres minor and deltoid muscles as well as give cutaneous innervation to the upper lateral aspect of the arm. (less)

A 14-year-old boy fell off his skateboard, hitting his right elbow on the cement sidewalk. Because he was experiencing severe elbow pain, his mother took him to the hospital. On examination, the orthopedist concluded that his elbow was fractured. Radiographs revealed that there was a separation of the epiphysis of the medial epicondyle of the humerus. He noted that there was numbness and tingling of the medial part of the palm and the medial one and a half digits. Which of the following statements best describes the most likely nerve injury?

The answer is ulnar nerve injury. This injury commonly occurs where the nerve passes posterior to the medial epicondyle of the humerus. The nerve was probably sharply compressed resulting in bruising and crushing of nerve fibers as the epiphysis was separated away from the humerus. The medial palmar surface and medial one and a half digits would be affected. Median nerve injury is incorrect. The median nerve passes anterior to the elbow and is separated from the medial epicondyle of the humerus by the common flexor origin. Injury of this nerve at the elbow results in paralysis of most of the muscles in the anterior compartment of the forearm and most intrinsic muscles of the thumb. This injury produces an "ape hand" and the "hand of benediction." These signs were not detected during the physical examination. Injury to the deep branch of the radial nerve is incorrect. This nerve passes anterior to the elbow and the lateral epicondyle. This is the motor nerve to the extensor muscles in the posterior compartment of the forearm. There would be no sensory loss because the deep radial nerve is a purely motor nerve. Injury to the superficial branch of the radial nerve is incorrect. This nerve supplies sensory fibers the dorsal aspect of the hand and dorsal surface of the lateral digits. Injury to the musculocutaneous nerve is incorrect. This nerve is rarely injured because it is protected by the biceps brachii. Just proximal to the elbow, the nerve emerges from the cover of the biceps as the lateral cutaneous nerve of the forearm. It does not supply sensation to the fingers. (less)

Your 12-year-old patient fell on his elbow and now has severe pain and deformity at the medial epicondyle of the humerus. A CT image shown above shows a displaced fracture (arrows). Based on this radiograph, you would most likely be concerned about damage to which of the following nerves?

The answer is ulnar. The ulnar nerve passes posterior to the medial epicondyle of the humerus and is often injured in elbow fractures.

A 21-year-old man injures his right arm in an automobile accident. Radiographic examination reveals a fracture of the medial epicondyle of the humerus. Which of the following nerves is most likely injured as a result of this accident?

The answer is ulnar. The ulnar nerve runs down the medial aspect of the arm and behind the medial epicondyle in a groove, where it is vulnerable to damage by fracture of the medial epicondyle. Other nerves are not in contact with the medial epicondyle. (less)

A 31-year-old patient complains of sensory loss over the anterior and posterior surfaces of the medial third of the hand and the medial one and one-half fingers. He is diagnosed by a physician as having "funny bone" symptoms. Which of the following nerves is injured?

The answer is ulnar. The ulnar nerve supplies sensory fibers to the skin over the palmar and dorsal surfaces of the medial third of the hand and the medial one and one-half fingers. The median nerve innervates the skin of the lateral side of the palm; the palmar side of the lateral three and one-half fingers; and the dorsal side of the index finger, the middle finger, and one-half of the ring finger. The radial nerve innervates the skin of the radial side of the hand and the radial two and one-half digits over the proximal phalanx. (less)

Lesion of the long thoracic nerve causes _____________.

The answer is winged scapula. The long thoracic nerve innervates the serratus anterior. Paralysis of serratus anterior causes the medial border of the scapula to pull away from the thoracic wall - winged scapula. (less)

The victim of an automobile accident has a destructive injury of the proximal row of carpal bones. Which of the following bones is most likely damaged?

The answer is triquetrum. The proximal row of carpal bones consists of the scaphoid, lunate, triquetrum, and pisiform bones, whereas the distal row consists of the trapezium, trapezoid, capitate, and hamate bones. (less)

Choose the appropriate lettered structure in the radiograph of the bones of the hand (see figure). Fracture of which bone may cause paralysis of the flexor digiti minimi and opponens digiti minimi muscles?

The answer is C. The hook of the hamate provides attachment for the flexor digiti minimi brevis and opponens digiti minimi muscles. Therefore, its fracture may cause paralysis of these muscles. (less)

Choose the appropriate lettered site or structure in this transverse magnetic resonance imaging through the middle of the palm of a woman's right hand (see figure) that matches the following description: The patient is unable to abduct her middle finger because of paralysis of this structure.

The answer is D. This is the second dorsal interosseous muscle, which abducts the middle finger.

Your 67-year-old female patient who fell on her hand has been diagnosed with a Colles fracture. This refers to which of the following?

The answer is a fracture of the distal radius in which the distal fragment is angulated dorsally A complete transverse fracture within the distal 2 cm of the radius in which the distal fragment is angulated dorsally is referred to as a Colles fracture. (less)

You are treating a 37-year-old male wrestler who dislocated his glenohumeral joint. He now also has some loss of sensation over the lateral side of his proximal arm. You are concerned that he may have sustained damage to which of the following nerves?

The answer is axillary. The axillary nerve is subject to damage during glenohumeral joint dislocations because it winds around the surgical neck of the humerus.

A 10-year-old boy falls off his bike, has difficulty in moving his shoulder, and is brought to an emergency department. His radiogram and angiogram reveal fracture of the surgical neck of his humerus and bleeding from the point of the fracture. Which of the following nerves is most likely injured as a result of this accident?

The answer is axillary. The axillary nerve runs posteriorly around the surgical neck of the humerus and is vulnerable to injury such as fracture of the surgical neck of the humerus or inferior dislocation of the humerus. The other nerves listed are not in contact with the surgical neck of the humerus. (less)

Your 12-year-old male patient is interested in a career in boxing and wants to learn exercises that specifically strengthen his "boxer's muscles" which is most important when reaching or punching anteriorly. What is this muscle?

The answer is serratus anterior. The serratus anterior is known as the boxer's muscle.

A 21-year-old man celebrating his birthday gets a little carried away with his friends and starts a bar fight. He is stabbed with a knife that severs the roots of C5 and C6 of the brachial plexus. Which of the following muscles is likely to be paralyzed?

The answer is infraspinatus. In Erb-Duchenne paralysis (or upper trunk injury), the nerve fibers in the roots of C5 and C6 of the brachial plexus are damaged. The infraspinatus, a lateral rotator muscle, is innervated by the suprascapular nerve (C5 and C6). All the other muscles, including the flexor carpi ulnaris, palmar interossei, adductor pollicis, and palmaris brevis muscles, are innervated by the ulnar nerve (C8 and T1). (less)

Your 45-year-old patient is showing weak hand pronation, especially in initiating the movement. You suspect a problem with the pronator quadratus. Which of the following is incorrect about this muscle?

The answer is it can be palpated from the dorsal aspect with deep pressure. The pronator quadratus cannot be palpated from any side of the forearm.

You are examining a patient with pain in her sternoclavicular joint and you suspect osteoarthritic changes in this joint. Which of the following is not correct about it?

The answer is it is a weak joint that is frequently dislocated. The SC joint is an extremely strong joint that virtually never dislocates in adults. The clavicle breaks before the joint dislocates. (less)

A 27-year-old pianist with a known carpal tunnel syndrome experiences difficulty in finger movements. Which of the following intrinsic muscles of her hand is paralyzed?

The answer is lateral two lumbricals and opponens pollicis. The median nerve innervates the abductor pollicis brevis, opponens pollicis, and two lateral lumbricals. The ulnar nerve innervates all interossei (palmar and dorsal), the adductor pollicis, and the two medial lumbricals. (less)

A 65-year-old man is brought to the emergency room after being attacked in his office by a disgruntled co-worker. The attacker reportedly used a long, narrow-bladed letter-opener to inflict multiple stab wounds to the man's back. Physical examination shows a puncture wound in the posterior axillary fold. The patient presents with weakness in extension, adduction, and medial rotation of his arm. Which of the following muscles is most likely cut in this injury?

The answer is latissimus dorsi. The axilla is a large, pyramidal space between the side of the chest and the upper part of the brachium. Its major importance is as a passageway from the root of the neck to the upper limb. The axilla is demarcated by four walls: anterior, posterior, medial, lateral. The posterior wall is composed of the latissimus dorsi, teres major, and subscapularis muscles. The posterior axillary fold forms the palpable lower margin of the wall and is composed of the latissimus dorsi and teres major. The subscapularis is not part of the posterior axillary fold. Damage to the latissimus dorsi would severely hinder adduction, extension, and medial rotation of the arm. Pectoralis minor is incorrect. The pectoralis minor contributes to the formation of the anterior wall of the axilla. The pectoralis major forms the bulk of the anterior wall and creates the noticeable anterior axillary fold. Levator scapulae is incorrect. The levator scapulae muscle passes out of the neck to attach onto the superior angle of the scapula. It is far removed from any of the walls of the axilla and does not contribute to rotation and adduction of the arm. Serratus anterior is incorrect. The serratus anterior lies against the thoracic wall and forms much of the medial wall of the axilla along with the thoracic wall. It fixes the scapula to the thoracic wall and has no affect on rotation of the arm. Teres minor is incorrect. The teres minor is located immediately above the teres major. However, it is not a component of the posterior wall of the axilla and normally contributes to lateral rotation of the arm. (less)

A 21-year-old woman walks in with a shoulder and arm injury after falling during horseback riding. Examination indicates that she cannot adduct her arm because of paralysis of which of the following muscles?

The answer is latissimus dorsi. The latissimus dorsi adducts the arm, and the supraspinatus muscle abducts the arm. The infraspinatus and the teres minor rotate the arm laterally. The serratus anterior rotates the glenoid cavity of the scapula upward, abducts the arm, and elevates it above a horizontal position. (less)

"Pronator teres syndrome" is a condition in which one of the following nerves is excessively compressed where it passes between the two heads of the pronator teres muscle. Which of the following nerves is entrapped?

The answer is median nerve. Each of the five terminal branches of the brachial plexus (musculocutaneous, median, ulnar, radial, and axillary nerves) passes through a muscular or osseofascial tunnel at some point in its distribution, where it may be subject to entrapment in a tunnel syndrome. The pronator teres muscle arises via two heads, one from the medial epicondyle of the humerus and the other from the coronoid process of the ulna, with a tendinous arch connecting them. The median nerve exits the cubital fossa and enters the forearm by passing between these heads, where it may be unduly compressed in a pronator teres syndrome. This condition would influence much of the median nerve territory in the forearm plus the entire median nerve territory in the hand. Deep branch of the radial nerve is incorrect. The radial nerve descends from the arm into the cubital fossa, where it divides into superficial and deep branches. The deep branch of the radial nerve pierces the supinator muscle, winds around the proximal end of the radius within the substance of that muscle, and passes into the deep posterior compartment of the forearm as the posterior interosseous nerve. The nerve may be entrapped within the supinator, resulting in a supinator syndrome. Such a condition would affect the deeper, more distal extensor muscles arising in the forearm and some sensory areas in the wrist joints. Deep branch of the ulnar nerve is incorrect. The ulnar nerve enters the hand superficial to the flexor retinaculum, runs through a groove between the pisiform and hook of the hamate (Guyon canal), and divides into superficial and deep branches at the base of the hypothenar eminence. The deep branch curls deeply there and enters the deep lying adductor/interosseous compartment in the palmar aspect of the hand. Compression of the ulnar nerve in Guyon canal may cause a Guyon tunnel syndrome, which affects the hypothenar muscles, medial two lumbricals, all interossei, adductor pollicis, and a large sensory area on both palmar and dorsal sides of the hand. Superficial branch of the ulnar nerve is incorrect. The superficial branch of the ulnar nerve does not enter a tunnel and is not subject to a tunnel syndrome. This nerve supplies the palmaris brevis muscle but is mostly cutaneous across the palmar and dorsal aspects of the medial third of the hand. Musculocutaneous nerve is incorrect. This nerve penetrates the coracobrachialis muscle, supplies the three flexor muscles in the anterior compartment of the arm, and continues into the forearm as the lateral cutaneous nerve of the forearm. Entrapment of the nerve within the coracobrachialis is rare. (less)

An 11-year-old boy falls down the stairs. A physician examines a radiograph of the boy's shoulder region (see figure). If the structure indicated by the letter B is fractured, which of the following structures is most likely injured?

The answer is posterior humeral circumflex artery. Fracture of the surgical neck of the humerus occurs commonly and damages the axillary nerve and the posterior humeral circumflex artery. (less)

A 74-year-old man complains of pain in his right hand and fingers when he works with his hands for a while. Thorough testing reveals insufficient blood flow into the deep palmar arch. Occlusion of which of the following arteries is the most likely cause of this condition?

The answer is radial artery. The radial and ulnar arteries anastomose to form the superficial and deep palmar arches. However, the superficial palmar arch is formed mainly by the ulnar artery, whereas the deep palmar arch is formed primarily by the radial artery. Thus, the insufficiency in this case is most likely due to reduced flow in the radial artery. Posterior interosseous artery is incorrect. The posterior interosseous artery branches off the common interosseous artery, which is derived from the ulnar artery in the proximal forearm. The posterior interosseous artery supplies the posterior compartment of the forearm. This artery does not reach the wrist and does not contribute blood to the palmar arterial arches. Ulnar artery is incorrect. The ulnar artery provides the primary supply into the superficial palmar arterial arch. The deep palmar arch is formed primarily by the radial artery, and it is the reduced flow of the radial artery most likely present in this patient. Anterior interosseous artery is incorrect. The anterior interosseous artery branches off the common interosseous artery, which is derived from the ulnar artery in the proximal forearm. The anterior interosseous artery supplies the anterior compartment of the forearm as well as the distal posterior forearm via its posterior terminal branch. This artery does reach the wrist, but it does not contribute blood to the palmar arterial arches. Inferior ulnar collateral artery is incorrect. This artery branches off the brachial artery just above the elbow. It passes distally across the anterior side of the medial epicondyle of the humerus to form a collateral connection with the anterior ulnar recurrent branch of the ulnar artery. This vessel is far removed from the hand. (less)

You are working in the ED treating a 34-year-old patient who was in a knife fight and has a wound under his right axilla. He seems unable to fully abduct his arm above horizontal. When you ask him to press against the wall you notice that he has a "winged scapula". Which muscle is likely paralyzed?

The answer is serratus anterior. The serratus anterior holds the medial border of the scapula against the thoracic wall. When paralyzed, the scapula projects posteriorly producing the "winged" appearance. (less)

A patient comes in with a gunshot wound and requires surgery in which his thoracoacromial trunk needs to be ligated. Which of the following arterial branches would maintain normal blood flow?

The answer is superior thoracic. The superior thoracic artery is a direct branch of the axillary artery. The thoracoacromial trunk has four branches: the pectoral, clavicular, acromial, and deltoid. (less)

As a result of chronic stress associated with an intense high school weight-lifting program, a 15-year-old boy suffers an avulsion fracture of the greater tubercle of the humerus. In the ER, he displays difficulty initiating abduction of the upper limb. Which of the following muscles was involved in this fracture?

The answer is supraspinatus. The greater tubercle of the humerus is the insertion site of three (of the four) rotator cuff muscles: supraspinatus, infraspinatus, and teres minor. Avulsion of this structure could result in detachment of any of these rotator cuff muscles, depending upon the size and scope of the fracture. However, the wrestler is unable to initiate abduction of the upper limb, which implies damage to the supraspinatus muscle. Long head of biceps brachii is incorrect. This muscle originates from the supraglenoid tubercle of the scapula and passes between the greater and lesser tubercles of the humerus, in the intertubercular (bicipital) groove. Detachment of the tendon of this muscle causes the biceps brachii to bulge in the anterior arm. Avulsion of the biceps brachii muscle is not related to the greater tubercle of the humerus. Long head of triceps is incorrect. The long head of the triceps brachii muscle originates from the infraglenoid tubercle of the scapula and inserts on the olecranon process of the ulna. It would not be involved in avulsion of the greater tubercle of the humerus. Subscapularis is incorrect. The fourth rotator cuff muscle, subscapularis, inserts onto the lesser tubercle of the humerus, so it would not be directly involved with this avulsion injury. Infraspinatus is incorrect. The infraspinatus muscle does insert onto the middle aspect of the greater tubercle of the humerus; however, damage to this muscle would result in weakness in external rotation at the shoulder joint, not the problems with abduction seen in this patient. (less)

A 55-year-old woman was riding her bicycle along a gravel path with her husband. Suddenly, she lost her balance and fell on her outstretched upper limb. She told her husband, a physician, that she heard a cracking sound when she hit the ground. He observed that her shoulder slumped inferomedially. He also noted that a medial fragment of her clavicle was elevated and that the lateral fragment was depressed. He concluded that she had sustained a typical fracture of the clavicle. He made a sling for her limb with his t-shirt. Which one of the following statements is likely incorrect pertaining to clavicular fractures?

The answer is the fracture usually involves rupture of the coracoclavicular ligament. This strong ligament usually is not ruptured in a clavicular fracture. The fracture was located at the junction of the lateral and intermediate thirds is incorrect. This is the most common site of clavicular fractures because it is the weakest part of the bone. It is most likely the fracture would occur here. The sternocleidomastoid muscle elevated the medial portion of a fractured clavicle is incorrect. This is a true statement. A clavicular fracture in very young children may be a "greenstick" fracture is incorrect. This is a true statement. In clavicular fracture, the lateral fragment is typically pulled medially is incorrect. This is a true statement because of adductor muscles such as pectoralis major. (less)

The posterior cord of the brachial plexus is formed by the ________________.

The answer is the posterior divisions of all 3 cords. Each trunk gives rise to an anterior and a posterior division. All 3 posterior divisions unite to form the posterior cord. (less)

A 27-year-old man with cubital tunnel syndrome complains of numbness and tingling in the ring and little finger and back and sides of his hand because of damage to a nerve in the tunnel at the elbow. Which of the following muscles is most likely to be paralyzed?

The answer is two medial lumbricals. The ulnar nerve innervates the two medial lumbricals. However, the median nerve innervates the two lateral lumbricals, the flexor digitorum superficialis, the opponens pollicis, and the pronator teres muscles. (less)

A 56-year-old woman was stopped at a light when her car was rear-ended by another car. She had her right arm on the steering wheel, and the impact caused forced flexion at her elbow. Several months later, she comes to her physician complaining of numbness and a "pins and needles" sensation in her right little finger when she talks on the phone, rests her head on her right hand at work, or spends most of her day typing at work. She also notices the quality of her typing and her ability to play the violin have diminished. Which nerve is compressed at what location?

The answer is ulnar nerve in the elbow. Situations in which peripheral nerves are compressed or otherwise entrapped where they pass through narrow spaces ("tunnels") in muscles and/or osseo-fascial units are generally referred to as "tunnel syndromes." Such conditions may result in periodic or constant motor and/or sensory deficits. The ulnar nerve crosses the elbow in a narrow space between the olecranon process and the medial epicondyle of the humerus, on the posteromedial (ulnar) side of the joint. The ulnar nerve can be compressed between these bony landmarks or between the humeral and ulnar heads of the attachment of the flexor carpi ulnaris. Compression of the ulnar nerve within these areas leads to "cubital tunnel syndrome." The symptoms are exacerbated during events where flexion of the elbow narrows these passageways and compresses the ulnar nerve at the elbow joint. The patient's cubital tunnel syndrome would explain the paresthesia and numbness on the medial aspect of the hand and the diminished fine motor control of the intrinsic hand muscles. Her condition originated due to the forced flexion of the elbow in the motor vehicle accident, which compressed the ulnar nerve in the cubital tunnel. Ulnar nerve in the wrist is incorrect. The ulnar nerve can be compressed between the pisiform and hook of the hamate at the wrist in a condition termed "ulnar canal syndrome" or "Guyon tunnel syndrome." This entrapment syndrome presents with similar signs and symptoms as seen in this patient. However, the ability to flex the wrist would not be affected. In this patient, trauma to the wrist was not reported. Median nerve in the wrist is incorrect. The median nerve is often compressed at the wrist in "carpal tunnel syndrome" because this nerve travels deep to the transverse carpal ligament (flexor retinaculum of the wrist) within the carpal tunnel. This entrapment syndrome presents with paresthesia of the lateral fingers as well as an inability to oppose the thumb and a wasting of the thenar eminence. These symptoms were not reported in this patient. Median nerve in the elbow is incorrect. The median nerve is not usually compressed by forced flexion of the elbow because it lies loosely on the flexor surface of the joint deep to the bicipital aponeurosis. The median nerve can be compressed in the proximal forearm as it passes between the two heads of the pronator teres. However, the symptoms in this patient were due to forced flexion of the elbow and involve the ulnar nerve. Median nerve in the axilla is incorrect. The median nerve would not be damaged in the axilla without significant trauma. Damage to the median nerve in the axilla would lead to weakness in flexing the wrist, loss of pronation of the forearm, and wasting of the thenar muscles. However, these signs and symptoms were not reported. (less)

You are testing the integrity of a nerve after an injury in a patient by holding the proximal interphalangeal joint of the little finger in the extended position and asking the patient to flex her distal interphalangeal joint. Which nerve is being tested?

The answer is ulnar. The ulnar nerve innervates the tendons of the flexor digitorum profundus that flex the distal IP joints of the fourth and fifth fingers.


Kaugnay na mga set ng pag-aaral

NRSG 245 Cumulative Pharmacology Final

View Set

Chapter 4 Wiley Extra Practice Questions

View Set

NUR 240 EAQ - Health Promotion and Maintenance

View Set

TEAS: Atoms and the Periodic table

View Set

EXAM 4 | CH 54 | Pancreas & Biliary problems

View Set

Care 2 Exam 1 Practice Questions

View Set